AAFP OBGYN

¡Supera tus tareas y exámenes ahora con Quizwiz!

A 34-year-old female with type 1 diabetes since age 16 presents to your office as a new patient for diabetes care. She is currently using NPH and regular insulin and has a hemoglobin A1c of 9.0% today. She has not seen a doctor for several years, as she has been without medical insurance until recently. She used to take lisinopril (Prinivil, Zestril) for hypertension and lovastatin (Altoprev) for hyperlipidemia but stopped these medications when her prescriptions lapsed more than a year ago. Her family history is significant for premature coronary artery disease in her mother, who suffered her first myocardial infarction at age 42.The patient has never been pregnant, and mentions that she and her husband would like to have a child in the near future. She asks what she needs to do to maximize her chances of a healthy pregnancy.Appropriate advice would include which of the following? (Mark all that are true.) Glucose levels should be maintained as close to normal as possible (hemoglobin A1c <7.0%) Lisinopril should be restarted Lovastatin should be restarted Referral for a dilated eye examination is indicated

A and D Preconceptional care is imperative in women with preexisting diabetes. All women of childbearing potential who have diabetes need to understand the importance of optimal glucose control prior to conception, to minimize the risk of congenital malformations and other pregnancy complications (SOR C). The current recommendation from the American Diabetes Association is that hemoglobin A1c be maintained as close to normal as possible (<7.0%) prior to conception (SOR B).ACE inhibitors such as lisinopril, and statins such as lovastatin, are contraindicated in pregnancy and should ideally be discontinued prior to conception (SOR C). This patient can be switched to an antihypertensive medication that is considered safe for use in pregnancy.Expert consensus recommends that women with diabetes who are planning a pregnancy be evaluated for secondary complications such as cardiovascular disease, diabetic retinopathy, diabetic nephropathy, and diabetic neuropathy, and treated accordingly. This patient already has known hypertension and hyperlipidemia, and is at high risk for coronary heart disease on the basis of family history and the length of time she has had diabetes.

True statements regarding genital herpes during pregnancy include which of the following? (Mark all that are true.) Acyclovir (Zovirax) is safe to use during pregnancy In patients with active genital lesions at the time of labor, cesarean delivery is recommended to decrease vertical transmission In patients with a history of previous genital herpes, cesarean delivery is warranted even without active genital lesions at the time of labor For patients with a history of genital herpes, treatment with oral acyclovir can decrease the rate of recurrent disease in the last month of pregnancy For patients with a history of genital herpes, serial cultures of the cervix and perineum during the third trimester can help guide therapy

A, B, D Guidelines for the management of herpes in pregnancy are available from many sources; those from ACOG and the CDC are readily available and are updated as needed.The third National Health and Examination Survey (NHANES III) found that the prevalence of antibodies to herpes simplex type 2 among reproductive age women in the United States was 25.6% in the mid 1990s, an increase of nearly one-third over the previous 15 years (level of evidence 2). In this survey, the majority of those with evidence of prior herpes infection reported no history of symptoms. The prevalence of these antibodies appeared to decline in the following decade, to 20.9%.The most important issue surrounding herpes in pregnancy is that infection of the newborn can lead to a devastating systemic infection, including encephalitis and meningitis, with long-term sequelae and death. Therefore, efforts to prevent vertical transmission of this virus are important. Evidence of an acute herpes outbreak in a pregnant woman at term is a reason to plan cesarean delivery (SOR B), which dramatically decreases the risk of infection in the newborn. While the risk of transmission is highest with a woman's primary infection, there is still a significant risk to the neonate from vaginal birth with recurrent infection, so determining whether there was a previous infection would not be pertinent if the outbreak were noted at the time of labor.Investigators have shown that treatment with acyclovir during the last 4 weeks of pregnancy can significantly decrease the risk of a recurrence at the time of delivery, and thus decrease the need for cesarean section. Because acyclovir is safe and well tolerated in pregnancy, it is recommended by the CDC as an appropriate treatment for women with initial outbreaks during pregnancy or with severe recurrent episodes of genital herpes during pregnancy (SOR C). There is growing evidence that treatment with acyclovir from 36 weeks gestation to delivery in women with previous herpetic outbreaks can decrease the rate of recurrent herpes and thus lower the need for cesarean delivery (SOR B). Additionally, pregnant women can reduce their potential exposure to herpes simplex virus (HSV) through the use of latex condoms and avoidance of intercourse with partners suspected of having active genital herpes.Finally, the CDC guideline states that routine serial genital cultures for HSV during the third trimester are not indicated, as prophylactic cesarean section is not recommended at the time of delivery for those without active genital lesions.

True statements regarding the use of heparin in pregnant patients with venous thromboembolism include which of the following? (Mark all that are true.) Pregnant patients with a history of a transient, isolated venous thrombosis with no underlying thrombophilia may not require heparin prophylaxis during the antepartum period Patients who have an underlying thrombophilia and a strong family history of thrombosis are candidates for antepartum and postpartum heparin prophylaxis, even with a negative personal history of thrombosis Epidural anesthesia is contraindicated in women taking unfractionated low-dose heparin Patients with antiphospholipid syndrome and a history of thrombosis require adjusted-dose prophylactic anticoagulation In patients receiving twice-daily doses of low molecular weight heparin, epidural anesthesia should be withheld until 24 hours after the last dose of heparin

A, B, D, E Pregnant patients with a history of isolated venous thrombosis directly related to a transient, highly thrombogenic event (orthopedic trauma, complicated surgery) may be offered heparin prophylaxis or no prophylaxis during the antepartum period if an underlying thrombophilia has been excluded. However, they should be counseled that their risk of thromboembolism is likely to be higher than that of the rest of the population. Prophylactic warfarin should be offered for 6 weeks post partum (SOR C).Pregnant patients with a history of idiopathic thrombosis, thrombosis related to pregnancy or oral contraceptive use, or thrombosis accompanied by an underlying thrombophilia should be offered antepartum and postpartum low-dose heparin prophylaxis (SOR C). This does not include those who are homozygous for the factor V Leiden mutation, or heterozygous for both the factor V Leiden and the prothrombin G20210A mutations, or who have antithrombin-III deficiency. These patients should be treated with adjusted-dose unfractionated heparin to maintain a partial thromboplastin time 1.5-2.5 times normal, or with adjusted-dose low molecular weight heparin.The safety of epidural anesthesia in patients receiving twice-daily doses of low molecular weight heparin is questionable, making it advisable to wait until 24 hours after the last injection of heparin before giving an epidural. However, epidural anesthesia appears to be safe in women taking unfractionated low-dose heparin if the activated partial thromboplastin time is normal. Patients with antiphospholipid syndrome and a history of thrombosis require adjusted-dose prophylactic anticoagulation (SOR C).Warfarin derivatives cross the placenta. A skeletal embryopathy resulting in stippled epiphyses and nasal and limb hypoplasia can occur when warfarin is given between 6 and 12 weeks of gestation. Midtrimester exposure may result in optic atrophy, microcephaly, and developmental delay. Bleeding can occur in the fetus at any time, resulting in a high fetal loss rate. Because of these concerns, warfarin is contraindicated throughout pregnancy. It is, however, acceptable for women who are breastfeeding.

During the second stage of labor, measures associated with a decrease in instrumented or difficult delivery include which of the following? (Mark all that are true.) Allowing a period for the baby to rest and descend if the mother does not have the urge to push Discontinuing epidural anesthesia at the onset of the second stage The use of coached pushing as opposed to uncoached spontaneous pushing Initiation of oxytocin (Pitocin) augmentation at the onset of the second stage Placing the mother in the upright or lateral position for the second stage rather than the dorsal lithotomy position

A, D, E The second stage of labor begins with complete cervical dilation and ends with delivery of the baby. Over the past several years, studies of this segment of labor have increased our understanding of the effects of maternal position, epidural anesthesia, pushing techniques, and oxytocin.Several studies have evaluated whether pushing should begin with complete dilation or can be delayed when epidural anesthesia is used, a technique referred to as "rest and descend" or allowing the baby to "labor down." In the largest randomized, controlled trial, involving 1862 nulliparous women receiving epidural anesthesia, the study group waited at least 2 hours after reaching complete dilation to start pushing compared to immediate pushing in the control group. Difficult and instrumented deliveries were reduced in the group that delayed pushing, and the two groups had similar infant and maternal outcomes. As subsequent studies have supported these conclusions, the use of rest and descend is a SOR A recommendation.A Cochrane review of discontinuation of epidural analgesia at the onset of the second stage showed those receiving continuous anesthesia and those who discontinued the epidural had no differences with regard to rates of instrumented delivery, cesarean delivery, or neonatal outcomes. However, the patients whose anesthesia was stopped were significantly more likely to report inadequate pain relief during the second stage (level of evidence 2).Several smaller studies have evaluated "coached," or closed-glottis, Valsalva pushing techniques against "noncoached," or spontaneous, pushing efforts during the second stage. One study randomized 320 nulliparas without an epidural to a coached and noncoached second stage. The second stage was found to be 13 minutes shorter in the coached group, but there were no changes in assisted vaginal delivery rates, the incidence of cesarean delivery, or abnormal fetal outcomes. Based on the aggregate of these small studies, there does not appear to be a preferred method of pushing at the current time (SOR B).Based upon a single randomized, controlled trial there is evidence to support initiation of oxytocin augmentation in nulliparas receiving epidural anesthesia at the start of the second stage. The study randomized 226 nulliparas with an epidural and no previous oxytocin during labor to either start oxytocin at the onset of the second stage or to receive no oxytocin. The group that received oxytocin had lower rates of assisted vaginal delivery and cesarean delivery, with similar infant outcomes.Finally, physician preferences for maternal positioning may increase rates of difficult delivery, as noted in a Cochrane review of 19 studies involving 5764 patients. This report noted that the upright or lateral position for pushing and delivery was associated with a decreased duration of the second stage, decreased rates of assisted vaginal delivery, reduced patient reports of severe pain during the second stage, and lower rates of abnormal fetal heart rate patterns (SOR A).

Risk factors for placental abruption include which of the following? (Mark all that are true.) Maternal cigarette smoking Maternal cocaine use Maternal blunt trauma Preeclampsia Chronic hypertension

All of the above Maternal cigarette smoking and cocaine use are well-recognized risk factors for placental abruption. Maternal blunt trauma is associated with fetal mortality of 3.4% to 38%, primarily from placental abruption, maternal shock, and maternal death. Leading causes of maternal blunt trauma in pregnancy include motor vehicle accidents, domestic violence, and falls.Chronic hypertension is the main risk factor for placental abruption. Preeclampsia, which can develop with or without a history of chronic hypertension, is also a risk factor for placental abruption.

True statements regarding fish consumption during pregnancy include which of the following? (Mark all that are true.) No fish should be eaten because of the risk of mercury poisoning Large predator fish such as shark, mackerel, and swordfish should be avoided entirely Raw seafood should not be consumed Moderate amounts of shrimp and catfish are considered safe Consuming smaller fish tends to be safer than consuming larger fish

B, C, D, E Fish and shellfish can be an important part of a healthy diet. Fish and shellfish contain high-quality protein and other essential nutrients, are low in saturated fat, and contain omega-3 fatty acids. A well-balanced diet that includes a variety of fish and shellfish can contribute to heart health and children's proper growth and development.However, nearly all fish and shellfish contain traces of mercury. For most people, the risk from mercury in fish and shellfish is not a health concern, but some fish and shellfish contain higher levels of mercury that may harm the developing nervous system in fetuses and young children. The risks from mercury depend on the amount of fish and shellfish eaten and the levels of mercury in the fish. Therefore, the Food and Drug Administration (FDA) and the Environmental Protection Agency (EPA) advise women who may become pregnant, pregnant women, nursing mothers, and young children to avoid some types of fish and to eat fish and shellfish that are lower in mercury (SOR C).Larger fish (swordfish, shark, king mackerel, and tilefish) pose the greatest risk to a fetus or small child because these fish have lived longer and have had more time to accumulate mercury. Other types of fish and shellfish may be eaten in the amounts recommended by the FDA and EPA.All uncooked fish and shellfish should be avoided during pregnancy (SOR C). Raw seafood is more likely to contain bacteria and parasites than cooked seafood. In addition, many shellfish filter large quantities of water through their shells, leading to increased accumulations of bacteria and viruses.

You see a 27-year-old gravida 2 para 2 on her second postpartum day after a normal vaginal delivery. When you examine her, she has a temperature of 38.2°C (100.8°F), but it was as high as 38.6°C (101.5°F) in the first 24 hours after delivery. She has no respiratory symptoms or urinary symptoms. Her uterus is mildly tender. She has no allergies.Which of the following antibiotic regimens would be appropriate for this patient? (Mark all that are true.) Intravenous ampicillin plus gentamicin Intravenous clindamycin (Cleocin) plus gentamicin Intravenous cefazolin Intravenous cefoxitin Oral amoxicillin/clavulanate (Augmentin)

B, D, and E Postpartum endometritis is generally considered synonymous with puerperal fever. It may be diagnosed in a febrile postpartum patient who has no evidence of another infection, such as a urinary tract infection or pneumonia. Uterine tenderness, foul smelling lochia, and a high WBC count support the diagnosis, but are not essential for making the diagnosis. Cultures are not necessary since the usual etiology is polybacterial, including anaerobes. The standard definition used for reporting puerperal fever is an oral temperature of at least 38.0°C on any 2 of the first 10 days post partum, or 38.7°C in the first 24 hours post partum (U.S. Joint Commission on Maternal Welfare).Typical rates of postpartum endometritis are 1%-3% after vaginal birth and 10%-30% after cesarean section. In addition to cesarean delivery, risk factors include duration of labor, number of vaginal examinations, duration of ruptured membranes prior to delivery, and anemia.A Cochrane review demonstrated that antibiotic prophylaxis at the time of cesarean section reduces the risk of postpartum endometritis by about 60%. Ampicillin and first-generation cephalosporins are equally effective (SOR A). Using multiple antibiotics for prophylaxis is not more effective.Another Cochrane review showed that intravenous clindamycin plus gentamicin, the traditional gold-standard therapy, is appropriate, as no other regimen has been shown to be more effective. If another antibiotic regimen is chosen, it should be one with activity against penicillin-resistant anaerobes such as Bacteroides fragilis (SOR A). Among the regimens listed, ampicillin plus gentamicin and cefazolin do not have activity against β-lactam-resistant anaerobes. Once-daily regimens of clindamycin plus gentamicin were as effective as the usual thrice-daily dosing. Oral amoxicillin plus clavulanate has been used successfully for treatment of postpartum endometritis after vaginal delivery.

You recently diagnosed essential hypertension in a 26-year-old female. Efforts at lifestyle modification have failed to lower her blood pressure sufficiently, and you determine that antihypertensive therapy is warranted. She is agreeable to this, but mentions that she hopes to become pregnant within 6 months. She and her husband have been using a barrier method of contraception.Appropriate antihypertensive choices at this time would include which of the following? (Mark all that are true.) Methyldopa Lisinopril (Prinivil, Zestril) Labetalol (Trandate) Atenolol (Tenormin)

A and C Given this patient's reproductive plans, it is crucial to choose an antihypertensive medication that is compatible with pregnancy. Both methyldopa and labetalol are commonly used during pregnancy for control of essential hypertension (SOR B). Methyldopa is FDA pregnancy category B, and labetalol is pregnancy category C. Calcium channel blockers are also acceptable.Atenolol is a β-blocker that should be avoided during pregnancy because of its association with fetal growth restriction (SOR B).ACE inhibitors should not be used in patients who are pregnant or are planning to become pregnant. Lisinopril carries a black box warning from the FDA (category X) regarding use during pregnancy because of the risk of fetal harm.Once the patient becomes pregnant, there is no benefit from treating mild hypertension (<150-160/100 mm Hg) during the pregnancy.

A 34-year-old gravida 4 para 3 sees you for a first prenatal visit. She is at 16 weeks gestation based on the dates of her last menstrual period. She has struggled with depression and has been suicidal in the past, but has been stable on sertraline (Zoloft), 50 mg daily. She asks you about the risks and benefits of continuing her antidepressant throughout pregnancy.Appropriate advice would include which of the following? (Mark all that are true.) Some studies have shown a slight increase in omphalocele and ventricular septal defects with first-trimester use of sertraline but the absolute risk is small Women who stop taking an antidepressant because of pregnancy have higher rates of morbidity, hospitalization, and suicide attempts Some studies have found an increased rate of persistent pulmonary hypertension in babies of women taking SSRIs There is no risk of withdrawal from sertraline as there is with some other SSRIs

A, B, C Whether to start or discontinue an antidepressant in pregnancy can be a difficult decision. Risks and benefits must be weighed and the patient must be involved in making the decision.There is no antidepressant completely free from fetal risk, and there is no FDA category B antidepressant. Bupropion used to be FDA category category B but was changed to category C based on rabbit studies that found an increased risk of skeletal abnormalities and decreased fetal weight.SSRIs are commonly prescribed in pregnancy. However, paroxetine is category D in pregnancy because it is associated with major congenital anomalies and cardiac defects; it should be avoided in pregnancy. First-trimester exposure to sertraline has been associated with omphalocele (odds ratio, 5.7; 95% CI, 1.6 to 20.7; 3 exposed subjects) and septal defects (odds ratio, 2.0; 95% CI, 1.2 to 4.0; 13 exposed subjects), but absolute risks are small.A large cohort study and a case control study of 337 women whose infants had persistent pulmonary hypertension of the newborn (PPHN) found that SSRI use in the second half of pregnancy was associated with an increased risk of PPHN. The increased risk of PPHN was similar among all SSRIs. SSRI use in pregnancy has also been associated with a neonatal withdrawal syndrome that can include convulsions, irritability, abnormal crying, and tremor. There is a risk of a withdrawal syndrome with all SSRIs, including sertraline.

Epidural analgesia is associated with which of the following? (Mark all that are true.) A prolonged first stage of labor A prolonged second stage of labor An increased rate of operative vaginal delivery An increased rate of cesarean delivery Maternal fever 5-minute Apgar scores <7

A, B, C, E Epidural analgesia is a common and popular form of labor pain control which can theoretically provide 100% pain relief during labor. Several systematic reviews have shown that while epidural analgesia is relatively safe and is not associated with an increased risk of cesarean section, it is associated with some negative effects, including a prolonged second stage of labor, an increased risk of maternal fever, and an increased rate of operative vaginal delivery (SOR A).

True statements regarding the management of preeclampsia include which of the following? (Mark all that are true.) For women at high risk for preeclampsia, low-dose aspirin should be prescribed as a preventive measure after 12 weeks gestation Calcium supplementation of 1000 mg/day has been shown to reduce the incidence of preeclampsia in primigravidas Magnesium sulfate is the drug of choice for the prevention and treatment of seizures in women with severe preeclampsia or eclampsia Antihypertensive drug therapy is recommended for pregnant women with systolic blood pressures of 140-160 mm Hg or higher, or diastolic blood pressures of 85-90 mm Hg or higher Serum uric acid is a reliable screening test for preeclampsia

A, C Both low-dose aspirin therapy and daily calcium supplementation have been studied as preventive measures for preeclampsia but have not been shown to be beneficial in the general pregnant population, and are not recommended for primary prevention of preeclampsia (level A recommendation). However, in 2013 the American College of Obstetricians and Gynecologists (ACOG) published new guidelines suggesting the initiation of daily low-dose aspirin (60-80 mg), beginning late in the first trimester, for women with a previous history of early-onset preeclampsia and delivery before 34 weeks gestation, or a history of preeclampsia in more than one pregnancy. In 2014 the U.S. Preventive Services Task Force recommended aspirin at a dosage of 81 mg/day after 12 weeks gestation as a preventive measure for women at high risk for preeclampsia (USPSTF B recommendation). There is also evidence that calcium supplementation may be of value in reducing the severity of preeclampsia in women with low calcium intake.ACOG recommends antihypertensive drug therapy for women with preeclampsia and sustained systolic blood pressures of 160 mm Hg or higher or diastolic blood pressures of 110 mm Hg or higher. Hydralazine and labetalol are the antihypertensive drugs most commonly used in women with severe preeclampsia (SOR C). While the ACOG guidelines include calcium channel blockers such as nifedipine as an option, there is a theoretical concern that the use of magnesium sulfate in patients taking nifedipine could result in hypotension and neuromuscular blockade. Labetalol should not be used in women with asthma or heart failure. ACE inhibitors are contraindicated in pregnant women.There is not a good screening test for preeclampsia. The serum uric acid level has been used as an indicator of preeclampsia but lacks sensitivity and specificity (SOR B). However, an elevated serum uric acid level may identify pregnant women with chronic hypertension who have an increased risk of developing superimposed preeclampsia.

A married couple planning a pregnancy within the next year sees you for preconception counseling. The wife is a 25-year-old nonsmoker who works as a schoolteacher, has never been pregnant, and is African American. Her 28-year-old husband smokes ½ pack of cigarettes per day, works as an auto mechanic, and is Hispanic. After reviewing lifestyle issues, including diet and exercise, and the importance of folate supplementation, the wife asks about preconception genetic testing. Neither partner can recall any genetic or birth defect history in their families.Which one of the following would you recommend strongly for this couple prior to conception? Hemoglobin electrophoresis DNA testing for cystic fibrosis mutations Karyotyping for balanced translocation Hexosaminidase A levels Genetic counseling referral prior to any tests

A Preconception health care includes both health maintenance and preventive care for the mother (and father), along with appropriate lifestyle modification, immunization, and preconception screening and risk modification for a future pregnancy.Ethnic background and family history are key elements to formulating an approach to preconception genetic screening. A recent report from the CDC on preconception care stresses the importance of this in recommending additional counseling or testing for those with a family history or ethnic risk for inherited disorders (SOR C). A family history of cystic fibrosis (CF), for example, would lead to specific carrier testing for CF that would involve searching for a specific defective gene; current carrier testing in the population usually tests for the most common 25-30 gene abnormalities in a specific area, and thus can miss 10% or more of carriers in specific populations. Currently, CF genetic testing is recommended for all Caucasians and should be offered to all other ethnic groups (SOR C).For African-American patients, the possibility of being a carrier of the sickle cell anemia recessive gene is about 1 in 11, so hemoglobin electrophoresis is recommended for this group. Because patients who are Hispanic, Mediterranean, Middle Eastern, or Southeast Asian can be carriers of α- or β-thalassemia genes, hemoglobin electrophoresis is recommended also, especially if the patient's mean red cell volume is below 80 fL (SOR C).In the absence of a significant family or ethnic history, there is no specific reason to consider parental karyotyping or screening for Tay-Sachs disease via hexosaminidase A levels in parents who are not of Ashkenazi Jewish ancestry. Some patients may ask to be referred to a genetic counseling team, but in the absence of any ethnic or family history to support this referral, the yield would be low.

A 26-year-old gravida 2 para 1 at 38 weeks gestation presents to labor and delivery with 4 hours of decreased fetal movement, leakage of fluid, and some mild uterine contractions. She denies bleeding, dysuria, headaches, and vision changes. The pregnancy has been normal, including appropriate fundal height measurements, blood pressures, and laboratory results; she had ultrasonography at 8 weeks gestation to confirm dates, and normal anatomic ultrasonography at 20 weeks gestation. Her first child was a 3450-g (7 lb 6 oz) male born at 40 weeks gestation after a 12-hour uncomplicated labor.She is found to have 2 contractions in 10 minutes, and a cervical examination shows that she is 5 cm dilated, 50% effaced, and at -1 station with a vertex presentation. Obvious rupture of the membranes with clear fluid is noted. A fetal heart tracing reveals a baseline of 140 beats/min with moderate variability, occasional early decelerations, and recurrent variable decelerations with a nadir of 90 beats/min, no late decelerations, and no accelerations.True statements regarding this patient's situation include which of the following? (Mark all that are true.) Amnioinfusion could decrease her risk of cesarean delivery for an abnormal fetal heart rate pattern She should be repositioned Oxytocin (Pitocin) should be started now She requires treatment for uterine tachysystole She has a category III fetal heart rate tracing

A and B Fetal heart rate monitoring is categorized by a three-tiered system: category I is considered normal, and category III is abnormal, characterized by a lack of variability and associated with an increased risk of fetal acidemia. Category II includes those fetal heart rate tracings that do not meet the criteria for categories I or III. This patient has a category II strip based on the presence of both recurrent variable decelerations and moderate variability. The presence of moderate variability suggests that fetal acidemia is not present. Management of recurrent variable decelerations includes maternal positioning to relieve presumed umbilical cord compression. Amnioinfusion has been shown to alleviate recurrent variable decelerations and thereby decrease the rate of cesarean sections performed because of a worrisome fetal heart rate pattern.This patient has normal uterine activity, as she has five or fewer contractions in 10 minutes averaged over a 30-minute period. Uterine tachysystole is defined as more than five contractions in 10 minutes averaged over 30 minutes. Because she has normal uterine activity, and has a category II tracing, oxytocin is not indicated and may be detrimental at this time.

True statements regarding genetic screening include which of the following? (Mark all that are true.) All pregnant women should be offered serum marker screening for neural tube defects and trisomies 21 and 18 The risk of Down syndrome increases in a gradual linear fashion up until a maternal age of 35 years The optimal timing for chorionic villus sampling is 10-12 weeks gestation The optimal timing for amniocentesis is before 15 weeks gestation

A and C Down syndrome (trisomy 21) is the most recognized genetic cause of mental retardation, with an estimated prevalence of 9.2 cases per 10,000 live births in the United States. The risk of a woman having a child with Down syndrome increases in a gradual, linear fashion up until age 30 and increases exponentially thereafter. The risk of having a child with Down syndrome is 1/1300 for a 25-year-old woman, 1/365 at age 35, and 1/30 at age 45.Pregnant women should be offered serum marker screening for neural tube defects and trisomies 18 and 21. Most physicians use the mid-trimester maternal serum screen, which measures human chorionic gonadotropin (hCG), unconjugated estriol, and α-fetoprotein levels at 15-20 weeks gestation (optimal timing is 16-18 weeks gestation). The maternal serum screen is approximately 65% sensitive for aneuploidy and 95% specific.In some centers, fetal nuchal translucency can be measured by ultrasonography combined with maternal serum analyte levels (i.e., free hCG and pregnancy-associated plasma protein A). This testing can be performed at 10-14 weeks gestation. Women with an increased risk for aneuploidy should be offered prenatal diagnosis by amniocentesis or chorionic villus sampling. Factors associated with increased risk include the following: age >35 at delivery with a singleton pregnancy, or >32 with twins fetus with a major structural anomaly identified by ultrasonography ultrasound markers of aneuploidy (including increased nuchal thickness) a previously affected pregnancy a known translocation, chromosome inversion, or aneuploidy in either parent a positive maternal serum screen Amniocentesis is usually offered between 15 and 20 weeks gestation and is associated with a 0.5% risk of spontaneous abortion. Amniocentesis performed between 11 and 13 weeks gestation is associated with significantly higher rates of pregnancy loss and complications. Chorionic villus sampling is performed at 10-12 weeks gestation and carries a 1.0%-1.5% risk of spontaneous abortion.

A 40-year-old primigravida who recently moved to your area sees you for an initial visit at 18 weeks gestation. Her pregnancy is the result of an intrauterine insemination, and she had an ultrasound evaluation at 6 weeks gestation. Your medical assistant alerts you that her initial blood pressure is 164/104 mm Hg, with a repeat reading of 166/105 mm Hg 10 minutes later. The patient is on no medications other than prenatal vitamins and feels well. Although she knows her blood pressure was measured at her previous physician's office, she is unaware of the readings and does not remember being told she has high blood pressure. Which of the following would be appropriate in the management of this patient at this time? (Mark all that are true.) A 24-hour urine collection to assess for proteinuria Lisinopril (Prinivil, Zestril), 20 mg daily Atenolol (Tenormin), 50 mg daily Nifedipine (Procardia), 60 mg daily Bed rest

A and D Hypertension that is detected prior to 20 weeks gestation or is known to have existed prior to the pregnancy is considered chronic hypertension during pregnancy. It is classified as mild (systolic blood pressure 140-159 mm Hg or diastolic blood pressure 90-109 mm Hg) or severe (systolic >160 mm Hg or diastolic >110 mm Hg). Because this patient is at less than 20 weeks gestation and has a blood pressure approaching the severe range, it is appropriate to treat her blood pressure to decrease the risk of maternal stroke. Chronic hypertension is also a risk factor for the development of preeclampsia. It is therefore appropriate to order a 24-hour urine collection to establish a baseline amount of protein. Elevated proteinuria above this baseline level later in pregnancy could indicate the development of superimposed preeclampsia. Certain illicit substances such as cocaine and methamphetamine can cause hypertension in addition to causing adverse fetal outcomes, and a hypertensive pregnant woman should be screened for these drugs. Chronic hypertension affects fetal growth, and it is recommended that these patients have baseline anatomic ultrasonography at 18-20 weeks gestation followed by serial ultrasound evaluations to monitor fetal growth.ACE inhibitors and angiotensin receptor blockers (ARBs) are contraindicated in all trimesters of pregnancy. ACE inhibitors have been associated with teratogenicity, including severely underdeveloped calvarial bone, renal dysgenesis, and pulmonary hypoplasia, as well as adverse fetal outcomes including intrauterine growth restriction, fetal death, neonatal renal failure, oligohydramnios, anuria, and neonatal death. ARBs have been associated with renal abnormalities, dysmorphia, and stillbirth. While atenolol, a pure β-blocker, has been associated with fetal growth restriction, labetalol, a combination α- and β-blocker, is considered safe in pregnancy. Nifedipine is the most commonly studied calcium channel blocker in pregnancy and is also an appropriate treatment of hypertension. There is no evidence that bed rest confers any benefit in these patients.

True statements regarding the use of intrapartum electronic fetal monitoring in low-risk, uncomplicated pregnancies include which of the following? (Mark all that are true.) It increases the risk for cesarean section compared with intermittent auscultation It reduces the risk for assisted delivery by forceps It reduces the risk for perinatal death compared with intermittent auscultation Intermittent auscultation at appropriate intervals appears equivalent to electronic fetal monitoring with regard to detecting fetal compromise

A and D Intermittent auscultation of the fetal heart rate appears to be equivalent to continuous electronic fetal monitoring with regard to detecting fetal compromise in low-risk, uncomplicated pregnancies. Auscultation every 30 minutes is recommended for low-risk patients in the active phase of labor, with the interval decreased to every 15 minutes in the second stage of labor (SOR C). Randomized, controlled trials comparing electronic fetal monitoring to intermittent auscultation have shown that electronic monitoring is associated with a higher rate of cesarean section and instrument deliveries, with no difference in the rate of perinatal deaths (level of evidence 1).

You see a 28-year-old gravida 2 para 1 at 25 weeks gestation. She has had mild to moderate vaginal bleeding with some cramping for the past 2 days. Her blood type is O-negative. Ultrasonography shows a marginal placenta previa and no signs of abruption.Appropriate management would include which of the following? (Mark all that are true.) A course of corticosteroids Admission to the hospital for monitoring and bed rest until term Planning for a cesarean section by 37 weeks, after assessing for fetal lung maturity A Kleinhauer-Betke test followed by an appropriate dose of anti-D immunoglobulin Frequent follow-up digital vaginal examinations to rule out preterm labor

A and D Placenta previa is a common finding (4%) with first and early second trimester ultrasonography. Marginal placenta previa is defined by the presence of the placental edge less than 2 cm from the internal os. The majority of marginal cases will resolve by term, and cesarean delivery is needed only if the condition persists (SOR A). Neonatal morbidity secondary to prematurity is a primary concern, and corticosteroids are recommended for bleeding caused by placenta previa between 24 and 34 weeks gestation (SOR A). While admission to the hospital for initial assessment is usually warranted, many women can be managed at home if the bleeding stops. Complete bed rest until term has not been shown to be effective (SOR A). While the risk of Rh alloimmunization after second-trimester bleeding is uncertain, many experts recommend giving anti-D immunoglobulin after performing a Kleihauer-Betke test to determine the appropriate dose (SOR C). Speculum examinations can be safely performed, but digital vaginal examinations should be avoided in patients with placenta previa, due to the risk of hemorrhage.

You are providing routine prenatal care to a healthy 21-year-old female. She is now at 18 weeks gestation. Ultrasonography at 13 weeks gestation confirmed her dates and was negative for nuchal translucency abnormality. She asks you when she will have her next ultrasound examination.True statements regarding this situation include which of the following? (Mark all that are true.) She should be offered ultrasonography at 18-20 weeks gestation to screen for structural abnormalities Because her size and dates have good congruence she does not need another study for dating purposes She should not have another study because of the potential harm from unnecessary repeat ultrasonography Ultrasonography near term has been shown to decrease the rate of postterm pregnancy

A, B No evidence directly links improved fetal outcomes with routine ultrasound screening. However, there is good evidence that early ultrasonography (before 14 weeks gestation) accurately determines gestational age (level of evidence 2), decreases the need for labor induction after 41 weeks gestation, and detects multiple pregnancies (level of evidence 1). Ultrasonography at 10-13 weeks gestation can measure nuchal translucency as a screening test for Down syndrome. Pregnant women should be offered ultrasonography to detect structural anomalies between 18 and 20 weeks gestation (SOR A). Diagnostic ultrasound exposure has not been proven to harm the mother or the fetus, but more research on its risks is needed.

True statements regarding perineal lacerations associated with vaginal delivery include which of the following? (Mark all that are true.) Antenatal digital massage of the perineum has been shown to decrease the incidence of perineal lacerations in nulliparas Use of a vacuum extractor rather than forceps is associated with a decrease in maternal perineal trauma Repair of perineal lacerations with absorbable synthetic sutures rather than catgut is associated with less short-term maternal pain Use of a continuous subcuticular suture technique rather than interrupted transcutaneous stitches is associated with more pain during the first 10 days after delivery

A, B, C Even if a restrictive approach to episiotomy is used, perineal lacerations occur during vaginal delivery. A 2006 Cochrane review of three randomized, controlled trials involving 2434 women supports the use of digital perineal massage, especially in nulliparas. In this group of women, perineal massage by the patient or her partner, as little as 1-2 times a week starting at 35 weeks gestation, was associated with a decrease in perineal trauma requiring suturing (NNT = 14) and a decrease in episiotomy (NNT = 23) (SOR A).Vacuum extractors and forceps both have advantages and disadvantages. While the vacuum extractor is associated with an increased incidence of cephalohematoma in the newborn, its use is associated with fewer maternal lacerations and less perineal trauma than forceps (NNT to prevent one severe maternal trauma by using a vacuum extractor rather than forceps = 11) (SOR A).Several suture materials can be used for perineal laceration repair; studies have now compared catgut to synthetic absorbable material such as polyglycolic acid for these lacerations. A Cochrane review of eight randomized, controlled trials involving 3642 women who required perineal repair following a vaginal birth showed that polyglycolic acid was associated with a significant reduction in pain during the first 3 days after delivery (NNT = 9), as well as for days 4-10 (NNT = 17) (SOR A).Several studies have compared a continuous subcuticular repair to interrupted transcutaneous stitches for lacerations of the perineum. A Cochrane review of four randomized, controlled trials involving 1864 women showed that there was significantly less pain during the first 10 days after the repair when the continuous subcuticular method was used (NNT = 14). No differences were seen in the use of pain medication, need for resuturing, or dyspareunia (SOR A).Based upon these findings, it is reasonable to encourage nulliparas to consider perineal digital massage at least 1-2 times per week starting at 35 weeks, use a vacuum extractor to reduce maternal trauma when planning assisted vaginal delivery, select a synthetic absorbable suture such as polyglycolic acid for perineal laceration repair, and do the repair with a continuous subcuticular stitch when possible in order to decrease short-term maternal pain.

A 28-year-old primigravida sees you for an initial prenatal visit. Her last menstrual period was 6 weeks ago and was normal. The pregnancy was unplanned but welcome. Her health history is significant for generalized epilepsy since childhood, well controlled with medication. Her last seizure was 3 years ago, and occurred after she inadvertently missed a few doses of her antiepileptic medication. She asks you about the implications of her seizure disorder for the pregnancy and wants to know if she should continue taking her medication. She hopes to breastfeed and asks if that will be possible if she is taking antiepileptic medication. Appropriate advice would include which of the following? (Mark all that are true.) Use of antiepileptic medications during pregnancy is associated with an increased risk of major congenital anomalies such as ventricular septal defect and cleft lip and palate Use of antiepileptic medications during pregnancy is associated with an increased risk of minor congenital anomalies such as dysmorphic facial features Seizures are unlikely to occur during pregnancy in women who have been free of seizures for 1 year when they become pregnant Use of a combination of antiepileptic medications is recommended during pregnancy to optimize seizure control Use of antiepileptic medication is a contraindication to breastfeeding

A, B, C Women who take antiepileptic medications benefit greatly from preconceptional counseling to outline the risks of pregnancy, both to the fetus and the mother, and to institute folic acid supplementation. The risk of congenital malformations in fetuses whose mothers took antiepileptic medications during pregnancy seems to result entirely from the medications themselves, and not from maternal epilepsy. Use of antiepileptic drugs during pregnancy appears to approximately double the risk of both major and minor congenital anomalies relative to the general population. The increased risk of congenital anomalies with older medications such as valproate and carbamazepine seems to be largely mediated by the folic acid deficiency that these drugs induce, although a recent guideline from the American Academy of Neurology (AAN) recommends against using valproate in pregnancy. It is not yet clear whether the newer antiepileptic medications are significantly less teratogenic than the older ones.Up to one-third of pregnant women with epilepsy appear to experience an increase in seizure frequency during pregnancy. This has to do with pregnancy-related changes in medication pharmacokinetics, as well as with reduced medication compliance.However, women with epilepsy can be counseled that being seizure-free for at least 9 months prior to the pregnancy appears to be associated with a high rate of freedom from seizures during pregnancy (84%-92%) (SOR B). The current recommendation is that pregnant women with epilepsy use a single antiseizure medication at the lowest effective dose (SOR C). Multi-drug therapy and higher doses are both associated with an increased risk of fetal malformations.Both the AAN and the American Academy of Pediatrics recommend encouraging women taking antiepileptic medications to breastfeed. These medications are excreted into breast milk to varying degrees, depending on their pharmacokinetics, so it is recommended that breastfed infants of women taking antiepileptic medications be monitored closely with respect to weight gain, sleep, and behavior (SOR C).

Contraindications to assisted delivery by vacuum extraction include which of the following? (Mark all that are true.) Prematurity (<34 weeks gestation) Unengaged head Suspected macrosomia Suspected fetal bleeding disorder Face presentation Maternal heart failure

A, B, C, D, E The exact gestational age at which a fetus is considered too premature for vacuum-assisted delivery is the subject of debate, but current guidelines recommend a gestational age of 34 weeks as the cutoff point (SOR C). The head should be engaged, and vacuum extraction should not be used if macrosomia or cephalopelvic disproportion is suspected. Suspected fetal bleeding abnormalities and bone demineralizing conditions such as osteogenesis imperfecta are also relative contraindications to vacuum delivery, due to the increased risk of fetal injury. Any fetal malposition, including face presentation, as well as uncertainty regarding fetal position, contraindicates vacuum application. Indications for vacuum extraction include inadequate maternal expulsive efforts, maternal exhaustion, fetal distress, prolonged second stage of labor, and the need to avoid voluntary expulsive efforts (e.g., in a mother with cardiac or cerebrovascular disease).

A 25-year-old recent immigrant from Ecuador presents to your office in November to establish care. In the course of taking her health history, you learn that she does not know whether she received any childhood immunizations and has no way of obtaining her childhood health records. She does not recall ever having had varicella. She has no known allergies and has had no significant illnesses. She works as a patient-care aide in a nursing home. She is newly married, is using oral contraceptives, and states that she hopes to become pregnant within the next 6-12 months. Her last menstrual period was normal and ended 3 days ago.Which of the following would be appropriate at this time? (Mark all that are true.) Inactivated influenza vaccine A varicella titer MMR Tdap A hepatitis B series

A, B, C, E Due to the increased risk of maternal and fetal morbidity and mortality, the inactivated influenza vaccine is recommended for all women who may or will be pregnant during influenza season and do not have contraindications to the vaccine. Given this patient's lack of documentation of childhood vaccinations, she should be offered Td now and after 4 weeks, and Tdap after 6-12 months. All pregnant women should be offered Tdap between 27 and 36 weeks gestation, regardless of prior vaccination history. If this patient becomes pregnant as planned, this could serve as her third dose of tetanus and diphtheria vaccine.This patient should also receive MMR, and either a varicella titer or a series of two varicella vaccines at least 3 months apart. In the vast majority of adults with no clear history of varicella, a titer will show that they are immune, so it is cost-effective to draw the titer initially and vaccinate if it is negative (level of evidence 3). This patient should be cautioned not to become pregnant within 4 weeks of receiving MMR and/or varicella vaccine, because both vaccines contain attenuated live virus and are, therefore, potentially teratogenic (SOR C).

A 33-year-old gravida 3 para 2 at 33 weeks gestation sees you for a follow-up visit. At her prenatal visit last week you noted that her fundal height was large for dates and that there seemed to be excess amniotic fluid. She had ultrasonography yesterday which confirmed polyhydramnios with an amniotic fluid index of 29 and a single deepest pocket of 9 cm. There were no obvious fetal malformations.Which of the following would be appropriate to tell this patient? (Mark all that are true.) Additional testing, including karyotyping, should be considered to rule out fetal anomalies Gestational diabetes should be ruled out She is at increased risk for complications, including perinatal death She is at increased risk for postterm pregnancy If she is very uncomfortable, some fluid could be removed periodically

A, B, C, E Polyhydramnios occurs in 1%-4% of pregnancies, with fetal anomalies identified in up to 50% of cases. The most common types of anomalies are those of the digestive, renal, and central nervous systems. Karyotyping and testing for infectious etiologies such as toxoplasmosis and cytomegalovirus may be considered (SOR C). Gestational diabetes is the most common maternal cause of polyhydramnios, accounting for about 20% of cases. Twin-twin transfusion is another common cause of polyhydramnios, which is not present in this case. Often no cause is identified.The risk of perinatal death in cases of polyhydramnios is high. One large observational study identified a risk of stillbirth of 43% when polyhydramnios was diagnosed in the second trimester and 16% when diagnosed in the third trimester, as in this case. Another observational study of fetuses with no identifiable anomaly showed an association of polyhydramnios with adverse outcomes, including low Apgar scores, perinatal death, and intensive care admissions. Polyhydramnios is also associated with an increased risk for preterm birth. Although anencephaly is associated with postterm pregnancy, that anomaly has been excluded in this patient.If the patient is very uncomfortable due to severe polyhydramnios, amnioreduction (removal of fluid) is an option. It carries a small risk of complications, principally premature rupture of membranes and placental abruption. However, amnioreduction has not been shown to reduce the rate of preterm birth.

Components of the Bishop score to measure cervical favorability for labor induction include which of the following? (Mark all that are true.) Cervical dilation Cervical effacement Fetal station Fetal presentation Cervical consistency

A, B, C, E The Bishop score is a systematic scoring system used to predict successful vaginal delivery after labor induction. A low score (usually defined as <6) is associated with a greater risk for cesarean section with labor induction, and current practice recommends using a cervical ripening agent prior to labor induction. A high score (usually defined as >8) is associated with a lower risk for cesarean section with labor induction (similar to spontaneous labor) (level of evidence 2). The Bishop score has five components: cervical dilation, cervical effacement, fetal station, cervical consistency, and fetal position. These are scored on a 3-point scale.Many clinicians use the modified Bishop score, which adds risk factors to the scoring system. This system adds 1 point for preeclampsia and 1 point for each previous vaginal delivery, and subtracts 1 point for primiparity, postterm gestation, and premature or prolonged rupture of membranes.

A 29-year-old gravida 3 para 2 at 30 weeks gestation comes to your office in November with symptoms of an upper respiratory infection. You prescribe symptomatic treatment, but she returns 1 week later feeling even worse than before. She says she has had a fever to 103°F and two episodes of shaking chills. She has also had a cough, and for the past 2 days it has produced thick, gray mucus. This morning she noticed streaks of blood in the mucus. She also has had significant dyspnea when walking around in her house. The patient smokes 1 pack of cigarettes per day, despite being counseled to quit. She has been taking oral iron because of anemia, but has had no other complications during her pregnancy.On examination she appears ill and is tachycardic, with a respiratory rate of 32/min after walking to the examination room. She has decreased breath sounds with crackles in her left lung base.Which of the following would be appropriate in your evaluation and management of this patient? (Mark all that are true.) A nasopharyngeal swab for influenza A and B A chest radiograph with abdominal shielding Assessment of oxygenation by either arterial blood gases or pulse oximetry Outpatient management with 10 days of a fluoroquinolone Inpatient management with a β-lactam and a macrolide intravenously

A, B, C, E The incidence of pneumonia in pregnancy is between 1.5 and 2.7 per 1000 pregnancies, which is similar to the rates in nonpregnant women of the same age. Risk factors for pneumonia during pregnancy include smoking, asthma, anemia, underlying lung disease, HIV infection, and cocaine use. Pneumonia in pregnancy is associated with an increased rate of miscarriage, preterm labor, and decreased birth weight.Causative agents for pneumonia during pregnancy are similar to those in the general population, with Streptococcus pneumoniae being the most common agent, followed by Haemophilus influenzae and Moraxella catarrhalis. Atypical agents, such as Mycoplasma and Chlamydia, were found to account for 10% of cases in one series. Influenza virus can cause pneumonia in pregnancy, and because rates of hospitalization for pregnancy-associated influenza equal those of patients over age 65, the CDC recommends influenza vaccine for all pregnant women during the months of October to April (SOR C).While trials specific to the management of pneumonia in pregnancy are lacking, American Thoracic Society criteria for inpatient versus outpatient management are supported by data (SOR B). These guidelines state that inpatient management should be provided for those with underlying pulmonary or other systemic illness, altered mental status, respirations over 30/min or temperature >100.5°F, a WBC count <4000/mm3 or >30,000/mm3, a PaO2 on room air <60 mm Hg or a PaCO2 >50 mm Hg, a creatinine level >1.2 mg/dL, or pleural effusion or multilobe involvement on a chest radiograph (appropriate in pregnancy if done with abdominal shielding). Additional tests that can aid in management include a nasopharyngeal swab for influenza A and B, and urine antigen testing for S. pneumoniae.Treatment is directed at the most likely etiologies. For bacterial pneumonia, outpatient treatment with an oral macrolide is appropriate for those who do not meet any of the above criteria. If hospitalization is required, use of a macrolide and a β-lactam antibiotic is appropriate. Fluoroquinolones should be avoided in pregnancy due to possible concerns about fetal toxicity. Supportive care, including hydration, supplemental oxygen, and management of bronchospasm with β2-agonists is also important.

A 24-year-old primigravida at 22 weeks gestation sees you because of a depressed mood. For the last 6 weeks, she has felt less and less like doing anything other than staying at home and lying in bed due to a lack of energy. She reports that she has been sleeping more than 12 hours a night for the last 2 weeks, and has stopped her daily piano playing, which she usually enjoys. She also has noted difficulty concentrating. She says that her husband is concerned about her. She denies suicidal thoughts or feelings, but does admit to feeling as if she has little hope of feeling better. Her mother had recurrent depression and is currently taking sertraline (Zoloft) with good results. The pregnancy has gone well up to this point.Which of the following would be appropriate for evaluation and management? (Mark all that are true.) Screening the patient for underlying illness, such as anemia, hypothyroidism, or diabetes mellitus Referral to a psychologist for interpersonal therapy Recommending an aerobic exercise program as the primary treatment Prescribing sertraline, 50 mg/day Scheduling a visit in 4 weeks for follow-up and routine prenatal care

A, B, D For women, the peak age of onset for depressive disorders is during the childbearing years. Studies have shown that pregnancy has a neutral effect on the onset of depression, but the rate is significantly higher during the postpartum period. The criteria for diagnosing major depression in pregnancy are the same as for nonpregnant patients, and this patient meets several of the DSM-IV criteria (more than four within a 2-week period are required, including depressed mood or loss of interest).The DSM-IV lists the following criteria: depressed mood most of the day markedly diminished interest or pleasure in all activities significant weight loss or gain, or change in appetite insomnia or hypersomnia psychomotor agitation fatigue or loss of energy feelings of worthlessness or inappropriate guilt diminished ability to think or concentrate recurrent thoughts of death or suicidal thoughts These symptoms must not be due to substance abuse or bereavement and must cause significant distress or impairment of function.Depression during pregnancy has been linked to an increased risk for preeclampsia, preterm delivery, and low birth weight. However, these findings are not consistent across all studies.Because some of the common complications of pregnancy can be associated with depressed energy level or mood, it is appropriate to evaluate pregnant women with these symptoms for evidence of anemia, thyroid disease, or diabetes mellitus.While a reasonable exercise program can be beneficial during pregnancy, it is not a specific treatment for depression or depressed mood, and it should not be recommended as a sole treatment in this patient. Additionally, because this patient meets the criteria for major depressive illness, reassurance with follow-up in 4 weeks is not appropriate.Treatment during pregnancy must balance risks to both the mother and the fetus against benefits, and individualization of the treatment plan is appropriate. Interpersonal therapy can be used alone or combined with antidepressant medication, or the medication can be used within an existing support structure (SOR C). For the specific situation of depression in pregnancy, significant guidance from large controlled trials is lacking.Finally, concerns about effects of antidepressant medication on the developing fetus are widespread. Fluoxetine has been on the market the longest of the SSRIs, and it has generally been considered the safest. However, there is growing information about an abstinence syndrome experienced by about 30% of all newborns exposed to SSRIs, consisting of high-pitched crying, tremors, and jitteriness. Perhaps more concerning is a case-control trial that showed a sixfold increase in the incidence of persistent pulmonary hypertension in newborns exposed to SSRIs in utero (level of evidence 2). These risks are still considered small and, in the case of the abstinence syndrome of short duration, must be balanced against the benefits for the pregnant woman, especially during the last 20 weeks of pregnancy. The association of paroxetine with cardiac defects has led to its classification by the FDA as a category D agent during pregnancy.

A 29-year-old patient who is 4 weeks post partum presents with a 1-day history of pain in her left breast. On examination her temperature is 37.2°C (99.0°F). You note localized redness, swelling, and tenderness of the left breast, but no fluctuance. The right breast is normal for a lactating woman.True statements about this patient's diagnosis and treatment include which of the following? (Mark all that are true.) She has lactational mastitis Staphylococcus aureus is a commonly isolated pathogen A fluoroquinolone would be appropriate antibiotic therapy A first-generation cephalosporin would be appropriate antibiotic therapy Breastfeeding should be discontinued during treatment

A, B, D Mastitis is an inflammatory condition of the breast that is usually associated with lactation. Its estimated frequency ranges from 2% to 33%. Staphylococcus aureus and coagulase-negative staphylococci are commonly isolated pathogens, with Escherichia coli and streptococci less frequently found. Clinical symptoms of mastitis include localized breast pain, redness, and swelling. Fever and malaise may also be present and indicate infection. If an abscess is present, a fluctuant mass may be found.Expression of milk is the mainstay of treatment, and there is general consensus that lactation should be continued to permit proper drainage of the breast (SOR C). Appropriate antibiotics include penicillinase-resistant penicillins and cephalosporins. Fluoroquinolones are not recommended during lactation.

True statements regarding HIV and pregnancy include which of the following? (Mark all that are true.) The Centers for Disease Control and Prevention (CDC) recommends routine opt-out prenatal HIV testing With appropriate interventions the mother to child transmission rate for HIV can be decreased from 25% to 2% Because of teratogenicity concerns, a single antiretroviral agent is preferable to a triple antiretroviral regimen for HIV infection in pregnancy Use of a fetal scalp electrode should be avoided if possible if an HIV-positive mother opts for attempted vaginal delivery Early rupture of the membranes and augmentation of labor are recommended to minimize the length of the second stage of labor if an HIV-positive mother opts for attempted vaginal delivery

A, B, D The CDC recommends universal opt-out prenatal screening for HIV. Repeat testing in the third trimester, before 36 weeks gestation, is recommended for women at high risk for HIV infection, including those living in areas with a high incidence of HIV, those with risk factors for transmission, and those with signs or symptoms of acute HIV infection. There is a rapid HIV test that can be used for women who present in labor with an unknown HIV status.Women diagnosed as HIV-positive during pregnancy should be informed that the risk of mother to child HIV transmission is estimated at 25%-30%, but that interventions such as antiretroviral therapy, cesarean section, and avoidance of breastfeeding can reduce this risk to less than 2% (level of evidence 1). Antiretroviral therapy should be given to the mother antenatally and at the time of delivery, and to the neonate for the first 4-6 weeks of life (SOR A).The optimal choice of antiretroviral therapy changes over time, and should be based on the most recent guidelines, as well as local patterns of viral resistance and viral genotyping. A triple-antiretroviral-therapy regimen is preferable for reducing mother to child transmission, and the risk of adverse events from the drugs is small.Elective cesarean delivery prior to rupture of the membranes is recommended at 38 weeks gestation for women with an HIV RNA load over 1000 copies/mL. Women who opt for a planned vaginal delivery should have their membranes left intact for as long as possible. The use of fetal scalp electrodes and fetal blood sampling should be avoided.

A 25-year-old gravida 2 para 1 at 27 weeks gestation is riding in the front passenger seat of a car when the driver swerves to miss an oncoming car and strikes a tree at about 30 mph. The woman is wearing a lap and shoulder belt, and the front air bags deploy. She is brought to the emergency department on a spine board. She is awake and alert, and complains of right shoulder and bilateral knee pain. She hasn't felt the baby move since the accident, about 40 minutes ago.True statements regarding the management of this patient include which of the following? (Mark all that are true.) Radiologic studies required to evaluate the mother should be done with abdominal shielding A maternal initial survey, with attention to airway, breathing, and circulation, should be carried out before assessment of the fetus The mother should remain immobilized in the supine position until the cervical spine can be evaluated radiographically Because of the potential for placental abruption and preterm labor, fetal monitoring for at least 4 hours is indicated A Kleihauer-Betke test can help assess for significant fetal-maternal hemorrhage but is not predictive of fetal outcome

A, B, D, E Trauma affects about 1 in 15 pregnancies and leads to hospitalization in about 1 out of 250 pregnancies in the United States. Motor vehicle accidents are the most common cause of trauma in pregnancy, followed by domestic violence and falls. Even in the absence of external injury to the mother, the fetus is at risk from placental abruption and other complications of trauma.Several maternal physiologic changes that occur during pregnancy make it more difficult to assess for maternal injury and hemorrhage. Plasma volume is increased by nearly 40%, and because of this change and others in the cardiovascular system, maternal blood pressure and pulse can remain normal despite significant blood loss. Additionally, after 20 weeks gestation, the uterus compresses the inferior vena cava and decreases venous return from the lower extremities when the mother is supine, making it important to always provide either a lateral tilt for the patient or to manually displace the uterus laterally off the great vessels to prevent hypotension.When evaluating a pregnant patient with trauma, the first step is to assess maternal stability with an initial survey of airway, breathing, and circulation. Once maternal respiratory and cardiovascular stability is ascertained, further evaluation of maternal injuries, as well as assessment of the fetus, can be undertaken. Supplemental oxygen, intravenous fluid (such as normal saline or lactated Ringer's solution), and evaluation of maternal injuries, including radiologic studies with abdominal shielding, are all appropriate.The workup of the pregnant patient should also include blood type and Rh factor, as trauma has the potential to cause fetal-maternal transfusion due to its effects on the uterus. The use of the Kleihauer-Betke test to help quantify the amount of any fetal-maternal transfusion is controversial; some expert groups recommend this as a routine part of the evaluation of trauma in pregnancy (SOR B, level of evidence 2), while others suggest the test be used only to quantify the amount of Rh-immune globulin (RhoGAM) the patient should receive (SOR C). For all patients who are Rh negative, RhoGAM should be administered even in the setting of minimal trauma.Another controversy surrounds the duration of fetal monitoring. Most sources suggest that after significant trauma beyond 20 weeks gestation, the fetus should be monitored for a period of time. Based on one prospective study, a period of 4 hours seems to be a reasonable minimum, with continued monitoring beyond that point if the patient has more than four contractions per hour, any vaginal bleeding, or abnormal laboratory values (SOR B).

In the active management of the third stage of labor, recommended interventions for the prevention of postpartum hemorrhage include which of the following? (Mark all that are true.) Starting oxytocin (Pitocin) upon delivery of the anterior shoulder Starting oxytocin upon delivery of the placenta Administration of oral prostaglandins Early clamping and cutting of the umbilical cord Constant, controlled traction on the umbilical cord to facilitate delivery of the placenta

A, B, E Postpartum hemorrhage, defined by the World Health Organization as the loss of at least 500 mL of blood in the first 24 hours after delivery, complicates approximately 4% of vaginal deliveries and is estimated to cause one-fourth of all maternal deaths related to childbirth. There are numerous maternal and labor-related factors that are known to increase the risk of postpartum hemorrhage, but many such hemorrhages occur unexpectedly. For that reason, active management of the third stage of labor, the interval between delivery of the baby and delivery of the placenta, has been widely studied and is increasingly being recommended for routine use to prevent postpartum hemorrhage, even in situations in which it is not anticipated.Prophylactic administration of oxytocin has been proven effective in preventing postpartum hemorrhage whether it is administered with the delivery of the anterior shoulder or after the delivery of the placenta (SOR B). A double-blinded, randomized, controlled trial focusing on the timing of oxytocin administration found that initiating oxytocin with the delivery of the anterior shoulder did not increase the likelihood that manual removal of the placenta would be needed, and was as effective as later administration of oxytocin in preventing postpartum hemorrhage.Oral prostaglandins such as misoprostol are not routinely recommended for use in preventing postpartum hemorrhage (SOR A). Misoprostol, whether given orally or rectally, has been shown to be less effective than injectable uterotonic agents such as oxytocin. Misoprostol can also cause unpleasant side effects, such as vomiting, shivering, and hyperthermia. A Cochrane review determined that oral or sublingual misoprostol is effective for reducing severe postpartum hemorrhage when compared to placebo, and that neither intramuscular prostaglandins nor misoprostol is preferable to conventional injectable uterotonics as part of the management of the third stage of labor, especially for low risk women.Continuous, controlled traction on the cord until the placenta is delivered is also recommended. Delayed umbilical cord clamping does not increase the risk of postpartum hemorrhage, and the American College of Obstetricians and Gynecologists now recommends delaying cord clamping in vigorous term and preterm infants for at least 30-60 seconds after birth (SOR A).

A 20-year-old white female comes to your office for her first prenatal visit. She quit smoking 2 weeks after a home pregnancy test was positive. Based on the date of her last menstrual period, she is at approximately 10 weeks gestation. Both she and her husband, who is also present, are happy about the pregnancy. She denies any past history of vaginal discharge, sexually transmitted diseases, or abnormal Papanicolaou smears. When you discuss the screening tests that are customary during prenatal care, both the husband and wife question the need for screening for sexually transmitted disease.You tell the patient and her husband that the U.S. Preventive Services Task Force recommends that she be screened for which of the following? (Mark all that are true.) Syphilis HIV infection Herpes simplex Bacterial vaginosis Chlamydial infection

A, B, E Prenatal care offers an opportunity for screening for infectious diseases that could have an impact on the mother or the developing fetus. Most of these illnesses are asymptomatic for periods of time, and the disease can be treated or measures can be taken to prevent passage to the newborn (e.g., immunization of the infant for hepatitis B). The Centers for Disease Control and Prevention has evaluated existing screening tests, with their known sensitivity and specificity, against the benefits and costs to the pregnant woman and fetus, and has made the following recommendations.All pregnant women in the United States should be screened for human immunodeficiency virus (HIV) infection as early in pregnancy as possible. Women who decline testing because they have had a previous negative HIV test should be informed of the importance of retesting during each pregnancy. Testing pregnant women and treating those who are infected are vital not only to maintain the health of the patient, but to reduce perinatal transmission of HIV through available antiretroviral and obstetrical interventions.A serologic test for syphilis should be performed on all pregnant women at the first prenatal visit. Women who are at high risk for syphilis, live in areas of high syphilis morbidity, or are previously untested should be screened again early in the third trimester (at approximately 28 weeks' gestation) and at delivery. Some states require all women to be screened at delivery. Pregnant women at risk for HBV infection also should be vaccinated. To avoid misinterpreting a transient positive HBsAg result during the 21 days after vaccination, HBsAg testing should be performed before vaccine administration. A positive result should prompt expedited administration of immunoprophylaxis to infants.All pregnant women should be routinely screened for Chlamydia trachomatis during the first prenatal visit. Women aged 25 years and those at increased risk for chlamydia (e.g., women who have a new or more than one sex partner) also should be retested during the third trimester to prevent maternal postnatal complications and chlamydial infection in the infant. Women found to have chlamydial infection during the first trimester should be retested within approximately 3 to 6 months, preferably in the third trimester.All pregnant women at risk for gonorrhea or living in an area in which the prevalence of Neisseria gonorrhoeae is high should be screened at the first prenatal visit for N. gonorrhoeae. Women aged <25 years are at highest risk for gonorrhea infection. Other risk factors for gonorrhea include a previous gonorrhea infection, other STDs, new or multiple sex partners, inconsistent condom use, commercial sex work, and drug use. Pregnant women found to have gonococcal infection during the first trimester should be retested within approximately 3 to 6 months, preferably in the third trimester.All pregnant women at high risk for hepatitis C infection such as those with a history of injection drug use or a blood or organ transplant prior to 1992 should be tested for hepatitis C antibodies at the first prenatal visit.Evidence does not support routine herpes simplex virus (HSV-2) serologic screening among previously undiagnosed women during pregnancy. In addition, the evidence does not support routine testing for bacterial vaginosis (BV) or Trichomonas vaginalis in asymptomatic pregnant women. In fact, the USPSTF recommends against screening for bacterial vaginosis in asymptomatic pregnant women at low risk for preterm delivery ( grade D recommendation)

A 24-year-old primigravida is at 41 weeks gestation. She is certain of the date of her last menstrual period, and her dates were confirmed by second-trimester ultrasonography.Options supported by evidence include which of the following? (Mark all that are true.) Induction of labor Expectant management with antenatal surveillance using nonstress testing and amniotic fluid indices Ultrasound Doppler velocimetry Encouraging coitus Membrane sweeping

A, B, E The incidence of postterm pregnancy varies based on the criteria used to define it; the incidence of pregnancies advancing to 42 weeks is about 5%-10%. Epidemiologic data demonstrates small but significant increases in adverse outcomes for fetus and mother.The weight of evidence suggests that routine induction of labor at 41 weeks has fetal benefit without incurring an increased risk of cesarean delivery (SOR B). However, this is not universally accepted, and there are some small studies showing increased cesarean section rates. Expectant management with antenatal surveillance is universally accepted despite evidence that it does not decrease perinatal morbidity or mortality (SOR C).Doppler velocimetry is of no benefit in the evaluation of postterm pregnancy and is not recommended unless the pregnancy is complicated by hypertension or concerns about intrauterine growth retardation (SOR B). A Cochrane review found that there is not enough evidence to demonstrate that women who engage in coitus at term are less likely to have postterm pregnancies. The theory is that prostaglandins in sperm stimulate labor, but this intervention is difficult to standardize. Another Cochrane review found that membrane sweeping reduces the need for labor induction, although the benefit should be balanced against the potential for bleeding and discomfort, and the number needed to treat to prevent one induction was 8.

A 28-year-old patient sees you for preconception counseling. She has never been pregnant and takes levothyroxine (Synthroid), 75 µg/day, for hypothyroidism. Her TSH levels have been in the normal range for the past 5 years.When counseling this patient, accurate advice would include which of the following? (Mark all that are true.) Untreated asymptomatic hypothyroidism is associated with pregnancy loss and decreased cognitive ability in the offspring The severity of cognitive disability in offspring is correlated with the severity of the maternal thyroid hormone deficit during pregnancy Appropriately treated maternal hypothyroidism is associated with an increased risk for adverse neonatal outcomes Women with hypothyroidism who become pregnant often need a lower dose of thyroid hormone replacement during the pregnancy Newborns of women with treated hypothyroidism often demonstrate a transient elevation in TSH in the first few days of life

A, B, E Untreated maternal hypothyroidism is associated with adverse neonatal outcomes, including pregnancy loss, prematurity, low birth weight, and poor cognitive function. There is a correlation between the severity of maternal hypothyroidism and the severity of cognitive disability in the offspring, which supports a causal relationship (i.e., the worse the deficit of thyroid hormone, the lower the intelligence quotient). Among a cohort of 20,000 women, with almost 500 having treated hypothyroid disease, there were no differences in adverse neonatal outcomes compared with women without hypothyroid disease. The dosage of thyroid hormone frequently needs to be increased during pregnancy. In a cohort study of 150 pregnancies in 114 women with primary hypothyroidism treated with levothyroxine prior to pregnancy, 70% required an increase in the dosage (SOR B).To some extent, the increased requirement may be related to interference of vitamins and calcium supplements with the absorption of thyroid hormone. A small cohort study of 32 hypothyroid women who were instructed to take thyroxine at least 4 hours before or after taking vitamins and calcium found that no significant change in dosing was required. No studies suggest a decreased dosing requirement in pregnancy.Many states screen infants for congenital hypothyroidism. Newborns of women with appropriately treated hypothyroidism may have false-positive elevations of TSH when tested at 25-120 hours after birth. Repeat TSH and free T4 levels are indicated in these infants.

A 24-year-old primigravida with an uncomplicated prenatal course is now at 37 weeks gestation. She has been reading several articles about management of labor, including information she found on the Internet. At today's visit she has a list of questions about episiotomy. Based on the evidence that you have reviewed, you and your partners have adopted a restricted use of episiotomy in your practice, rather than using it routinely.When she asks about this, you explain that restricting the use of episiotomy decreases which of the following? (Mark all that are true.) Posterior perineal trauma Anterior perineal trauma Healing complications at 7 days Blood loss at delivery The need for perineal suturing

A, C, D, E Episiotomy, used in a majority (62.5%) of deliveries in the United States in 1983, has been declining in use over the last 20 years. During this time, randomized trials have been performed that challenged many of the presumed benefits of the procedure, such as prevention of severe pelvic trauma or pain. Because an episiotomy can assist in speeding the delivery of a newborn during assisted vaginal delivery, or provide additional space for maneuvers to relieve shoulder dystocia, these studies have compared routine use of episiotomy (used in a majority of deliveries) with restricted use of episiotomy, done only in the presence of specific indications such as nonreassuring fetal heart tracings or shoulder dystocia.A Cochrane review of six randomized, controlled trials including more than 2400 women provides the data for answering this patient's questions. A restrictive approach to episiotomy has the following effects: a decreased rate of posterior perineal trauma (NNT = 10) an increased rate of anterior perineal trauma (NNH = 11) a decreased rate of healing complications at 7 days (NNT = 11) decreased blood loss at delivery (average decrease of 58 mL) less need for perineal suturing (NNT = 4.5) The Cochrane review also showed that there were no differences in reported patient pain scores, incontinence, or dyspareunia. Based on these findings, the authors concluded that a restrictive approach to episiotomy has significant benefits (SOR A).

A 26-year-old primigravida at 38 weeks and 3 days gestation presents to your office with a history of leaking clear fluid for the last 4 hours. Her pregnancy has been uncomplicated. Screening laboratory tests were normal, and she was GBS-negative at 36 weeks gestation. She denies having any contractions or vaginal bleeding, and states that the baby is moving.Which of the following would be appropriate when evaluating this patient? (Mark all that are true.) A sterile speculum examination A sterile digital vaginal examination Assessment of fetal heart tones Amniotic fluid index determination Blood pressure measurement

A, C, D, E The patient most likely has ruptured membranes and should be evaluated for term premature rupture of membranes (PROM), which occurs in 5%-10% of term pregnancies. The concern with PROM is that prolonged rupture of membranes is associated with an increased risk of both maternal and fetal infection. This risk is dramatically increased with early or frequent digital vaginal examinations. Therefore, it is recommended that digital vaginal examinations be avoided when possible (SOR B).To diagnose PROM when the history is unclear, a sterile speculum examination is recommended to detect pooling of amniotic fluid in the vaginal vault. Vaginal fluid pH should be checked with nitrazine paper (which will show an alkaline pH with amniotic fluid, but also with blood), and fluid should be allowed to dry on a slide and examined for ferning (SOR C).Sometimes abdominal ultrasonography is used to determine the amniotic fluid index, as a low amniotic fluid level could indicate occult rupture of membranes. In addition, maternal and fetal well being should be assessed, generally by maternal vital signs, including blood pressure, and fetal heart tones. ACOG recommends that induction with oxytocin is appropriate in patients at 34 weeks gestational age or beyond, with appropriate attention to group B Streptococcus prophylaxis.

A 24-year-old gravida 2 para 1 at 22 weeks gestation presents to your office with wheezing. She quit smoking before this pregnancy, and she has a history of asthma since childhood. She has managed her asthma adequately for several years with an albuterol (Proventil, Ventolin) metered-dose inhaler (MDI) that she has needed only about once a month. Two weeks ago, she began wheezing more than usual, and has since used her albuterol MDI 3-4 times a day to relieve her symptoms. This began after an upper respiratory infection, which is now better. She has had a dry cough, no fever, and no nausea or vomiting. She says the baby is moving normally.On examination, she has wheezing throughout both lungs without crackles, and has no findings to suggest pharyngitis, otitis media, or sinusitis. Her oxygen saturation on room air is 95%.True statements regarding this problem include which of the following? (Mark all that are true.) Rhinitis and gastroesophageal reflux are more common in pregnancy and can exacerbate asthma during pregnancy Theophylline is preferred for managing asthma during pregnancy because of its long history of safe use and low incidence of side effects For a patient with daily asthma symptoms, the preferred treatment is daily inhaled corticosteroids Studies of pregnant women treated acutely for asthma suggest that undermedication is rare during pregnancy Use of a rescue inhaler more than twice a week is an indication to start or intensify long-term asthma control therapy

A, C, E The National Asthma Education and Prevention Program (NAEPP) has issued several reports since 1993 that attempt to review the evidence for asthma care during pregnancy and present a clear approach for clinicians who manage pregnant patients with asthma. Recommendations from the most recent update, published in 2004, are available in a quick-reference format on the Internet.Population studies show that the prevalence of asthma during pregnancy is 3.7%-8.4%, which is similar to that of nonpregnant women of reproductive age. Most patients with asthma have improvement in their symptoms during pregnancy (69%) and a minority have worsening symptoms (31%). Exacerbations can be triggered by exposure to allergens or cold, upper respiratory infections, or symptoms that are common during pregnancy, such as rhinitis and gastroesophageal reflux. Additionally, cigarette smoking is associated with a higher incidence of exacerbations.Pharmacologic therapy for asthma is based on extrapolation from studies of nonpregnant patients, although there is some evidence from cohorts of pregnant women to help with assessment of the safety of specific medications. For acute symptoms, a rapid-acting β2-agonist inhaler is used, and albuterol has the most safety data to support its use (SOR B). If symptoms occur more than twice a week, necessitating use of albuterol, the asthma is no longer considered intermittent, but is persistent; for all types of persistent asthma, the cornerstone of therapy is inhaled corticosteroids, and in pregnancy, the agent with the most data to support its safety is budesonide (SOR B).According to the 2004 update of the NAEPP recommendations, if symptoms are not controlled with 6 puffs of budesonide per day, consideration can be given to adding a long-acting β2-agonist instead of a short course of oral prednisone. Theophylline, due to its small therapeutic range and known side-effect profile, is considered an alternative agent for use after the addition of inhaled corticosteroids and long-acting β-agonists.Although the recommendation is based on expert opinion (SOR C), one of the areas stressed by the NAEPP in the 2004 report was that it is safer for pregnant women to be treated with asthma medications than to have symptoms and exacerbations. Unfortunately, investigators who have evaluated ambulatory treatment of asthma exacerbations in pregnancy have found that, in general, pregnant women received corticosteroids for acute exacerbations only two-thirds as often as nonpregnant women of the same age and asthma severity; in addition, pregnant women were nearly three times more likely to have continued exacerbation symptoms 2 weeks after their initial treatment for acute asthma.

A 27-year-old female at 8 weeks gestation sees you for follow-up after an episode of vaginal bleeding. Her β-hCG levels are rising, and ultrasonography confirms a viable 8-week pregnancy with no apparent source of bleeding.Appropriate advice for this patient would include which of the following? (Mark all that are true.) She is at increased risk for miscarriage She should remain at bed rest for 1 week beyond the end of the bleeding episode The presence of heart activity in the embryo is reassuring She should take a progestogen to reduce the risk of miscarriage She has about a 1 in 8 risk of preterm delivery

A, C, E This is a patient with a viable pregnancy and first-trimester bleeding of undetermined etiology. Her chance of miscarriage is about 14%, or 1 in 7. If she were over 30 years of age, the risk of miscarriage would be about 27%. If the bleeding had occurred before 6 weeks gestation, the risk of miscarriage would be around 29%. There are only two etiologic factors for first-trimester bleeding that are accepted by all authors: parental balanced chromosomal translocations and uterine malformations. Other possible risks are smoking, excess caffeine intake, and asymptomatic hypothyroidism.Cardiac activity in the embryo is a reassuring sign, as it is associated with a pregnancy loss rate of 2% in women 35 years old and younger, and a loss rate of 16% in women older than 35.Cochrane reviews have found no evidence of effectiveness for bed rest to prevent miscarriage (SOR A), no evidence that vitamin supplements are effective in the prevention of miscarriage (SOR A), and no evidence to support routine use of progestogens to prevent miscarriage, although this review concluded that further research is needed to determine whether they are effective in women with a history of recurrent miscarriage.Patients with first-trimester bleeding of undetermined etiology are at increased risk for preterm birth, with a risk of about 12%, compared to about 6% in similar populations with no miscarriage history. However, studies to demonstrate improved outcomes with increased surveillance or preventive interventions are lacking.

A 30-year-old primigravida at 32 weeks gestation complains of generalized pruritus for the past week that has become intense enough to interfere with sleep. It is most intense on her palms and soles. There are no visible lesions on her skin, and she does not appear to be jaundiced. She has no other complaints. Fetal movement, heart rate, and fundal height are appropriate for gestational age.Laboratory FindingsTotal bilirubin............1.1 mg/dL (N 0.1-1.2)Albumin............3.6 g/dL (N 3.1-4.7)Total protein............6.5 g/dL (N 6.0-8.0)Alkaline phosphatase............120 IU/L (N 40-125)ALT (SGPT)............29 IU/L (N 10-39)AST (SGOT)............35 IU/L (N 4-42)Serum bile acids............21 µg/mL (N <12)True statements regarding the management of this patient include which of the following? (Mark all that are true.) She should be tested for hepatitis C She should have a liver biopsy She may benefit from treatment with ursodiol (Actigall) Cholestyramine (Questran) is first-line therapy Induction of labor is indicated as soon as she reaches term or when fetal lung maturity is confirmed

A, C, E This patient has intrahepatic cholestasis of pregnancy (ICP), evidenced by generalized pruritus and elevated serum bile salts. This condition results from impaired metabolism and excretion of bile acids from the maternal bloodstream. Genetic, hormonal, and nutritional factors appear to play a role in the pathogenesis. The incidence of ICP in the United States is about 7/1000 pregnancies. It is highest in Chile (over 10%) and Scandinavia (2%). More than 20% of women known to be infected with hepatitis C develop ICP, so testing is indicated. However, pathologic findings are not specific and a liver biopsy is rarely indicated.The principal risks of ICP are to the fetus and include spontaneous preterm delivery, stillbirth, and respiratory distress syndrome. Stillbirth occurs in up to 15% of untreated cases, with an autopsy typically showing signs of acute, but not chronic, anoxia. Stillbirth is not possible to predict, as it may occur within 2 days of reactive nonstress testing and with normal fetal kick counts. Stillbirth usually occurs after 37 weeks gestational age.The principal approach to management of ICP is delivery no later than 38 weeks gestation, and earlier if the case is severe and fetal lung maturity has been demonstrated (SOR C). Several medications have been used in the treatment of ICP. Ursodiol at a dosage of 1 g/day has been shown to provide symptomatic relief from pruritus and to decrease maternal serum bile salts (SOR B). Cholestyramine was considered appropriate treatment at one time. While it may provide symptomatic relief, it is inferior to ursodiol, does not decrease maternal serum bile salts, and may lead to deficiency of fat-soluble vitamins, including vitamin K (SOR B). Dexamethasone, which has been used along with both ursodiol and cholestyramine, has not been shown to have any benefit, and is associated with decreased birth weight.

You are called to attend the precipitous delivery of a patient you have not previously seen. When you arrive, the patient has just delivered the infant's head to the level of the nose, but further pushing yields no progress. You apply gentle traction on the infant's head, and deliver the remainder of the head, but not the anterior shoulder.Appropriate measures at this point include which of the following? (Mark all that are true.) Call for additional help Ask a nurse to apply fundal pressure while the patient pushes Have the patient flex and abduct her hips with assistance Provide supplemental oxygen to the mother Attempt to rotate the infant's anterior shoulder into an oblique position

A, C, and E Shoulder dystocia is defined as a delivery in which gentle downward traction fails to deliver the anterior shoulder, which has impacted against the symphysis pubis. The overall incidence of shoulder dystocia increases as fetal weight increases, with an incidence of 0.6%-1.4% in infants weighing between 2500 g and 4000 g, and 5%-9% in infants weighing between 4000 g and 4500 g. The incidence is increased in gestations complicated by diabetes, macrosomia, assisted vaginal delivery, abnormal pelvic anatomy, and a protracted active first or second stage of labor.Complications of shoulder dystocia can occur in both the mother and the infant. For the mother, there is an increased incidence of postpartum hemorrhage and lacerations of the genital tract, including fourth-degree lacerations; for the infant, there is an increased incidence of brachial plexus injuries, clavicle or humerus fracture, hypoxic brain injury, and death. This emergency should be managed rapidly and in a fashion that maximizes the likelihood of safe vaginal delivery while minimizing maternal and newborn complications.Several maneuvers have been proposed and studied for management of shoulder dystocia. For the family physician, an appropriate consideration, depending on the setting of the delivery, includes obtaining additional assistance for management of the mother, the baby, or both. The ALSO course has developed a mnemonic for management of shoulder dystocia (HELPERR) that describes an appropriate approach to this problem (SOR C; SOR B for patient roll):H = Help (call for additional help: nursing, anesthesia, or a cesarean section team)E = Evaluate for episiotomy (may be needed for maneuvers)L = Legs (McRoberts maneuver of flexing and abducting maternal hips)P = Pressure (suprapubic pressure to help disimpact the anterior shoulder)E = Enter the vagina (to attempt to rotate the anterior shoulder from behind into an oblique position, or to perform Woods screw or other maneuvers)R = Remove the posterior armR = Roll the patient to the all-fours positionWhile the order of these maneuvers can vary, and depending on the clinical situation may need to, each can reduce the impaction of the anterior shoulder.It is important to remember that fundal pressure can worsen the impaction and is thus contraindicated. Because the dystocia is created by the symphysis, intentional large episiotomies are not indicated unless required to perform the maneuvers above. Supplemental oxygen, stopping oxytocin, and other measures used for intrauterine fetal resuscitation do not have a role in immediate management.Additional support for the approach described here is found in the guideline released in 2005 by Great Britain's Royal College of Obstetricians and Gynecologists, which rates the clinical evidence for all of the above maneuvers, as well as several less common maneuvers. The guideline also includes an algorithm that allows for variation in management depending on the assessment of the clinician.

True statements concerning fetal alcohol syndrome (FAS) include which of the following? (Mark all that are true.) Maternal factors that increase the risk of FAS include age >30, a history of binge drinking, and lower socioeconomic status Because organogenesis is completed by the first trimester, recommendations against any alcohol consumption can be relaxed in the third trimester FAS most likely has a genetic transmission pattern because a woman who bears a child with FAS has a higher chance of bearing a second child with the same condition Facial anomalies are often the first distinguishing characteristic of an affected child

A, D Fetal alcohol spectrum disorders (FASD) are caused by the effects of maternal alcohol consumption during pregnancy. Fetal alcohol syndrome (FAS) is the most clinically recognizable form of FASD and is characterized by a pattern of minor facial anomalies, prenatal and postnatal growth retardation, and functional or structural central nervous system (CNS) abnormalities. The consequences are lifelong, and the behavioral and learning difficulties are often greater than the degree of neurocognitive impairment. Alcohol-related neurodevelopmental disorder also is a clinically recognizable diagnosis in the continuum of FASD, and describes the clinical outcome when the facial features typical of FAS are absent.Because no safe threshold of alcohol use during pregnancy has been established for any time during pregnancy, the CDC and the National Task Force on Fetal Alcohol Syndrome and Fetal Alcohol Effect recommend that women who are pregnant, planning a pregnancy, or at risk for pregnancy should not drink alcohol (SOR B). Women of childbearing age who are not pregnant should drink no more than seven drinks per week and no more than three drinks on any one occasion. Women who have had at least one child with FAS are at especially high risk for giving birth to a second affected child because of continued alcohol use, not because of genetic factors.Universal screening for alcohol use among all women of childbearing age might help identify women who drink more than recommended levels, as well as those who drink and might become pregnant. For women who screen positive for hazardous alcohol use or abuse, brief interventions that use time-limited, self-help, and preventive strategies to promote reductions in alcohol use in nondependent persons and that facilitate referral of dependent persons to specialized treatment programs are low-cost, effective treatment alternatives. Preconception counseling of women of childbearing age who are not using effective contraception and are at risk for alcohol use during pregnancy has been shown to be a promising method of prevention.

Current evidence supports which of the following methods for cervical ripening in labor induction? (Mark all that are true.) Dinoprostone (Cervidil) Evening primrose oil Castor oil Enemas Misoprostol (Cytotec) Foley bulb

A, E, F Labor induction is a common obstetric intervention. It is associated with higher cesarean section rates when performed in the absence of a ripe or favorable cervix. Many methods have been proposed to help ripen a cervix for labor induction, and cervical ripening is recommended if the Bishop score is less than 6. Nonpharmacologic ripening agents such as castor oil, hot baths, enemas, sexual intercourse, and breast stimulation are commonly suggested, but the evidence is lacking to support using them to improve cervical favorability. Cochrane reviews show that prostaglandins (dinoprostone) and Foley bulb use improve cervical favorability and increase the rate of delivery within 24 hours without increasing cesarean section rates (SOR A); misoprostol also increases the likelihood of delivery in 24 hours and decreases cesarean section rates (SOR A).

A 25-year-old gravida 2 para 2 presents for her 6-week postpartum visit. She reports that she plans to exclusively breastfeed her daughter until 6 months of age. The patient and her husband plan on becoming sexually active soon, and she requests that she be started on a contraceptive method.When discussing contraceptive choices with this patient, the potential for reducing breast milk production should be taken into consideration for which of the following? (Mark all that are true.) Combination oral contraceptives Progestin-only oral contraceptives Injectable medroxyprogesterone acetate (Depo-Provera) A subdermal progestin implant A copper-containing intrauterine device A vaginal contraceptive ring

A, F Based on World Health Organization guidelines, the use of combination hormonal contraceptives, including oral contraceptives, the hormonal contraceptive patch, and the vaginal ring, is generally not favored for women less than 6 months post partum who exclusively breastfeed, because of concerns that milk production may be diminished, thereby adversely affecting the growth of the infant. Guidelines from the American College of Obstetricians and Gynecologists acknowledge these concerns, but note that the use of combination oral contraceptives may be considered once milk flow is established.Studies have shown that in breastfeeding women less than 6 weeks post partum, progestogen-only contraceptives (including progestin-only pills), injectable progestin agents, and subdermal progestin implants do not adversely affect milk production or infant health and growth (level of evidence 2). Copper-bearing and levonorgestrel-releasing intrauterine devices can be inserted 4 weeks post partum regardless of whether the patient is breastfeeding.

Which of the following should raise suspicion that a pregnant woman may be the victim of domestic violence? (Mark all that are true.) Extreme concern for the well-being of the unborn child Late or insufficient prenatal care Poor weight gain Abnormal bleeding A partner who seems reluctant to leave the pregnant woman alone with the clinician

All None of the above situations is a definitive indicator of domestic violence, but the presence of one or more should raise the clinician's suspicions in this regard (level of evidence 2).Some research suggests that pregnancy is a risk factor for domestic violence, regardless of ethnicity, education, or socioeconomic level. It is therefore recommended that screening for domestic violence be incorporated into preconceptional and prenatal care (SOR C).The Department of Defense/VA clinical practice guideline for management of uncomplicated pregnancy recommends screening for domestic violence during routine prenatal care at weeks 8, 24, and 32 (SOR B), using the following questions: Within the last year, have you been hit, slapped, kicked, or otherwise physically hurt by someone? Since you've been pregnant, have you been hit, slapped, kicked or otherwise physically hurt by someone? Within the last year, has anyone forced you to participate in sexual activity?

Gestational diabetes is associated with which of the following outcomes? (Mark all that are true.) Fetal macrosomia Neonatal hypoglycemia Preeclampsia Neonatal hyperbilirubinemia An increased rate of cesarean delivery

All of the above Gestational diabetes mellitus (GDM) has been frequently associated with a wide variety of maternal and neonatal complications, including all of those listed. However, research has not consistently demonstrated an increase in perinatal mortality in pregnancies complicated by GDM, nor is it clear whether diagnosis and treatment of GDM improves perinatal outcomes (SOR C). Much of the controversy surrounding the diagnosis and management of GDM stems from the absence of large randomized, controlled trials on the subject.

You see a 26-year-old African-American for a routine 6-week postpartum visit. This was her first pregnancy, and was complicated by severe preeclampsia diagnosed at 28 weeks gestation. She had a small lower-extremity deep vein thrombosis at age 22, but no thrombotic issues during the pregnancy. She required oral antihypertensive therapy with labetalol (Trandate). She is doing well and is asymptomatic. Her blood pressure in the office is 128/78 mm Hg on no medication.True statements regarding her future risks and cardiovascular risk profile include which of the following? (Mark all that are true.) She is at increased risk for preeclampsia in future pregnancies Her past history of deep vein thrombosis is a risk factor for preeclampsia She is at increased risk for cardiovascular disease later in life She should be screened for protein C resistance (Factor V Leiden), antiphospholipid antibodies, homocysteinemia, and protein S deficiency

All of the above Women who have had preeclampsia are more prone to hypertensive complications in subsequent pregnancies. This increased risk is best established for those with a history of preeclampsia during their first pregnancy, with the risk increasing with earlier disease during the index pregnancy. When preeclampsia presents clinically before 30 weeks gestation the recurrence rate may be as high as 40%. A previous history of deep vein thrombosis also increases the risk.Several large epidemiologic studies have demonstrated an increased cardiovascular risk later in life in women with a past history of preeclampsia, and a history of preeclampsia is regarded as a risk factor for cardiovascular disease by the American Heart Association. These patients have been found to be at higher risk for hypertension, myocardial infarction, stroke, and heart failure.Women with early-onset severe preeclampsia may harbor metabolic abnormalities or risk factors associated with vascular thrombosis. These include activated protein C resistance (factor V Leiden), antiphospholipid antibodies, hyperhomocysteinemia, and protein S deficiency. Therefore, patients with a history of early-onset severe preeclampsia should be evaluated for evidence of prior thromboembolic diseases, and screened if this is found.

A 20-year-old primigravida at 32 weeks gestation presents to your office for her first prenatal visit with you. She was seen in another state until 18 weeks gestation, when the relocation of her family interrupted her prenatal care. She works as a custodian at a local school and smokes 1 pack of cigarettes per day. The patient gives you a copy of her prenatal record. You note that her estimated delivery date is based on an ultrasound examination at 10 weeks gestation, and that her prenatal screening tests were normal at all her visits. She has not had another ultrasound evaluation, did not have a serum maternal analyte screen for aneuploidy or neural tube defects, and has not been screened for gestational diabetes. She denies alcohol or drug use. Her prepregnancy weight was 58 kg (128 lb).Physical FindingsBlood pressure............132/82 mm HgWeight............64 kg (141 lb)Fundal height............26 cmFetal heart tones............145 beats/minShe has no peripheral edema, and her urine is negative for protein and glucose. A fingerstick hemoglobin level is 10.5 mg/dL (N 12.0-16.0, nonpregnant).True statements regarding this situation include which of the following? (Mark all that are true.) Anemia of pregnancy is a likely explanation for her poor weight gain The patient's smoking is a potential risk factor for intrauterine growth restriction (IUGR) Chronic hypertension is a risk factor for IUGR Ultrasonography should be done as soon as possible to assess fetal growth Uteroplacental insufficiency usually causes an asymmetric IUGR

B, C, D, E Intrauterine growth restriction (IUGR) is defined as a fetal weight below the 10th percentile for a specific gestational age, and is usually diagnosed by ultrasonography, although it can be detected on a physical examination if the gestational age in weeks exceeds the value of the fundal height in cm by 4 or more. Potential causes of IUGR can be divided into medical factors that lead to placental insufficiency, such as chronic hypertension or preeclampsia; maternal factors such as smoking, alcohol use, drug use, malnutrition, or living at or above an altitude of 5000 feet; and fetal factors that include congenital infections such as toxoplasmosis and cytomegalovirus, along with genetic anomalies such as trisomy 21.While IUGR can be suspected if a single ultrasound evaluation shows an estimated fetal weight below the 10th percentile, the accuracy of a single evaluation is limited due to the potential inaccuracy of pregnancy dates. If ultrasonography was performed in the first trimester to estimate dates, subsequent ultrasonography can more accurately diagnose poor fetal growth by using the results of the first evaluation.IUGR is often divided into symmetric and asymmetric types, but there is considerable overlap in the causes of these two types. However, placental insufficiency initially relatively spares fetal brain growth, leading to asymmetry (i.e., normal head growth with a lag in the increase of abdomen and femur measurements), while early insults such as infections or genetic abnormalities affect the fetus symmetrically from early in the pregnancy.If IUGR is noted on ultrasonography, or suspected by serial fundal height measurements and confirmed by ultrasonography, an investigation for possible reversible causes should begin, starting with evidence of maternal conditions such as hypertension, as well as drug exposure, including cigarette or alcohol use (SOR C). Sometimes, there are features found within the fetal scan that suggest a possible etiology, such as physical abnormalities consistent with genetic abnormalities or infections. If a possible reversible cause is found, it can be treated and the pregnancy monitored by serial ultrasonography.A relatively new addition to the monitoring methods for IUGR is Doppler ultrasound evaluation of the umbilical artery (often described as Doppler velocimetry). In cases of placental insufficiency, the low resistance of the placental blood vessels increases, yielding a detectable decrease in diastolic flow in the umbilical artery. In the setting of IUGR, ACOG recommends that Doppler ultrasonography be used to assess fetal status, as this has been shown to decrease perinatal mortality by identifying fetuses at increasing risk if not delivered (SOR C).

A 23-year-old primigravida sees you at 13 weeks gestation for her second prenatal visit. She states that her prenatal vitamins with iron make her constipated, and asks how important it is to take them.Appropriate advice would include which of the following? (Mark all that are true.) Pregnant women are routinely prescribed 90 mg of iron daily Iron deficiency anemia is associated with low birth weight Iron deficiency anemia is associated with preterm birth Iron absorption is enhanced in patients taking H2-blockers If she takes the iron supplement with meals it will probably bother her less, but less iron will be absorbed

B, C, E Iron supplementation in pregnancy is routine practice in the United States, with patients typically prescribed 30 mg daily (SOR C). Iron supplementation during pregnancy in women who are initially iron-replete decreases the risk of iron deficiency at term and postnatally. The main rationale for screening for anemia in pregnancy is that there is an association between iron deficiency anemia during pregnancy and low birth weight and preterm delivery. However, evidence regarding the benefit of treating iron deficiency anemia in pregnancy to reduce preterm birth and low birth weight is insufficient. One randomized, controlled trial showed a significant increase of 200 g in mean birth weight with iron supplementation compared to placebo, but no reduction in the rates of preterm birth or low birth weight.Gastrointestinal absorption of elemental iron is enhanced in the presence of an acidic gastric environment. Iron absorption is thus facilitated by the simultaneous intake of vitamin C. Iron is also absorbed better when taken on an empty stomach. Iron absorption is reduced by foods rich in tannates (e.g., tea) or phytates (e.g., bran cereal), and by medications that inhibit gastric acid production (e.g., antacids, proton pump inhibitors, histamine H2-blockers).Severe maternal anemia (hemoglobin <6.0 g/dL) has been associated with abnormal fetal heart tracings and is a reason for maternal transfusion for fetal indications. Most mild cases of maternal anemia (hemoglobin >9.0 g/dL) associated with iron deficiency will respond to oral iron supplementation over a period of several weeks.Iron supplementation during pregnancy does have some adverse effects. Principal among them are gastrointestinal side effects, which are reported by 10%-40% of women. The side effects are dose-related and may be reduced by postprandial administration, but this reduces absorption. Hemoconcentration may occur with hemoglobin concentrations above 13.0 g/dL, the usual physiologic range. The possible harm of hemoconcentration related to iron supplementation has not been established.

You see a 33-year-old female for a routine evaluation. She tells you she may be pregnant, and if she is would like to schedule her first prenatal visit with you next week. This would be her first pregnancy, and it was not planned. A review of her chart shows that she has had weight problems over the years but recently has not been actively dieting nor exercising. Her past medical history is otherwise unremarkable.On examination her height is 160 cm (63 in) and she weighs 79 kg (174 lb) (BMI 30.8 kg/m2). Her blood pressure is 110/65 mm Hg and her temperature is 37.0°C (98.6°F). A urine β-hCG in the office is positive. By dates she is at 8 weeks gestation.She wants to start "working on her weight." True statements regarding her weight include which of the following? (Mark all that are true.) She should not worry too much about weight gain during this pregnancy as long as she eats sensibly and walks daily Her BMI puts her at risk for many pregnancy complications, including hypertension, gestational diabetes, and preeclampsia The recommended weight gain for her during the pregnancy is 25 lb She is at higher risk for having a large for gestational age infant She has a higher-than-average probability of delivering by cesarean section

B, D ,E The patient has a BMI of 30.9 kg/m2, which places her in the obese category. Her target weight gain should be 11-20 lb for the pregnancy according to recommendations from the Institute of Medicine (SOR C). Her weight places her at increased risk for gestational diabetes, hypertension, preeclampsia, and eclampsia. She also has an increased risk for having a large for gestational age infant, increasing the likelihood of operative delivery (level of evidence 2). Underweight women who conceive with a BMI <18.5 kg/m2 are at higher risk for preterm delivery.

A 21-year-old primigravida at 12 weeks gestation, with no previous history of any drug allergies or urologic problems, presented 2 days ago for her first prenatal visit. She said she had no urinary tract or systemic symptoms. Today you receive a report that her urine culture is growing >100,000 colonies of a gram-negative rod, with identification and sensitivities to follow.True statements regarding this problem include which of the following? (Mark all that are true.) Because the patient is asymptomatic, no further treatment is recommended This condition is associated with an increased risk of pyelonephritis and low birth weight This condition can reliably be diagnosed by urine dipstick screening Treatment of this condition with appropriate antibiotics can decrease the rate of preterm birth Single-dose antibiotic therapy with amoxicillin or nitrofurantoin is not recommended during pregnancy

B, D, E This patient meets the criteria for asymptomatic bacteriuria of pregnancy: >100,000 colonies of a single organism on urine culture with no urinary symptoms. This condition occurs in approximately 10% of pregnancies, and when left untreated has been associated with increased rates of symptomatic cystitis, pyelonephritis, preterm labor, and low birth weight.Due to the relatively high prevalence and potential complications associated with asymptomatic bacteriuria, the U.S. Preventive Services Task Force recommends that all pregnant women be screened with a urine culture at 12-16 weeks gestation (SOR A). Urine dipstick screening is not recommended because it lacks sensitivity and specificity for this diagnosis.Women with asymptomatic bacteria in pregnancy have a twenty- to thirtyfold increased risk of pyelonephritis. Antibiotic treatment is associated with a decreased risk of ongoing asymptomatic bacteriuria (risk ratio [RR] 0.25, 95% confidence interval [CI] 0.14- 0.48), pyelonephritis (RR 0.23, 95% CI 0.13-0.41) and low birth weight babies (RR 0.66, 95% CI 0.49-0.89).Fluoroquinolones such as ciprofloxacin are not recommended in pregnancy because they may cause joint disease. Sulfonamides are not indicated because of concerns about teratogenicity in the first trimester and kernicterus later in pregnancy. The use of nitrofurantoin in pregnancy has been questioned because of concerns about teratogenicity in the first trimester and hemolytic disease in women with G-6-PD deficiency who are near term. Amoxicillin is often avoided because 30%-40% of Escherichia coli are resistant in some studies. Cephalexin may be a good choice, although it is not effective against enterococcus. While the optimal duration of treatment has not been determined, the Infectious Diseases Society of America recommends a 3- to 7-day course of therapy.

A 25-year-old primigravida at term presents to the hospital in early active labor, with cervical dilation of 2 cm. She would prefer not to use epidural anesthesia and asks about her other options for pain control.Which of the following would be accurate advice? (Mark all that are true.) Intravenous opioids provide pain control that is as effective as epidural anesthesia Hydrotherapy is effective for pain control during the first stage of labor Avoiding epidural anesthesia will decrease her risk of cesarean delivery Women who refuse epidural anesthesia report less satisfaction with their birth experience, due to increased pain Continuous labor support by a doula decreases both the need for analgesia and the likelihood of cesarean section

B, E Epidural anesthesia has been shown to be a safe and effective method of pain control in labor, but some women prefer to avoid its use. Providers should be familiar with the advantages and disadvantages of other options. While parenteral narcotics do provide a good method to control labor pain, they are not as effective as epidural anesthesia (SOR A). Several studies have demonstrated the effectiveness and safety of hydrotherapy, which decreases pain perception, decreases the need for other methods of pain control, and shortens labor, without increasing rates of cesarean section or maternal infection (SOR A). While there is some controversy over whether epidurals increase the risk of assisted vaginal delivery, studies have consistently shown no increased risk of cesarean section (SOR A). Maternal satisfaction with the delivery process is most strongly related to the doctor-patient relationship and involvement in the decision-making process. There is no strong correlation between pain control and satisfaction (SOR B). Continuous labor support from a trained doula has been shown to decrease the need for analgesia, decrease the cesarean section rate, and improve satisfaction with the labor experience (SOR A).

A 23-year-old patient comes to your office 4 weeks after the uncomplicated vaginal birth of her first child, with the chief complaint that she feels tired all the time. On further questioning, she describes significant emotional lability during the first week after delivery; she has continued to feel downcast, and worries about her ability to care for her child. She reports no history of personal or family depressive illness. Her infant is feeding and growing well, and now requires only one nighttime feeding.True statements regarding this condition include which of the following? (Mark all that are true.) The patient can be reassured that the problem will most likely be gone within another 4 weeks Appropriate screening for underlying medical conditions would include a urinalysis and an erythrocyte sedimentation rate The Edinburgh Postnatal Depression Scale is an appropriate screening tool to consider in this patient Pharmacologic therapy should be avoided because she is breastfeeding This condition is unlikely to recur in subsequent pregnancies

C Postpartum depression is relatively common, occurring in up to 10% of postpartum women. Maternal postpartum depression is disruptive to the family unit, placing the family at higher risk for paternal depression as well as marital discord, family violence, substance use and abuse, child abuse and neglect, failure to implement child safety and preventive measures, as well as poorer management of chronic health conditions in children. Postpartum depression is associated with both the early cessation of breastfeeding as well as the development of a neglectful environment, both of which have an adverse effect on infant development. Consequences of maternal depression include negative effects on cognitive development, social-emotional development, and behavior of the child.Sometimes it can be difficult to distinguish postpartum depression from "baby blues," a period of increased emotional lability, irritability, and fatigue that can begin in the first 24-48 hours post partum, but usually disappears within 2 weeks. Symptoms that persist beyond 2 weeks, including depressed mood, lack of pleasure, sleep disturbance, diminished concentration, feelings of guilt or worthlessness, loss of energy, and thoughts of death or suicide, are consistent with the DSM-IV diagnostic criteria for major depression. In those with a history of previous postpartum depression, the recurrence rate is more than 30%.During the postpartum period, complications of pregnancy and common medical conditions can create symptoms similar to those of depression. Screening for anemia and thyroid disease is appropriate because they are often seen in the postpartum period. Screening tests should include a CBC (or hemoglobin or hematocrit) and a TSH level or progressive thyroid panel.Both peripartum and postpartum depression screening are strongly recommended by the American Academy of Pediatrics. The Edinburgh Postnatal Depression Scale is a 10-item instrument to assess postpartum depression recommended by the U.S. Preventive Services Task Force. Multiple studies have confirmed its accuracy, and it can aid in screening for postpartum major depression (SOR A); one study in Australia found its sensitivity to be 100% with a specificity of more than 95% when using a cutoff of over 12 points. It can be completed by the patient while in the waiting room and then quickly scored to provide an assessment for the clinician during a postpartum visit. One shortcoming is that its results have not been verified in non-English-speaking populations. (This screening instrument can be downloaded at http://www.fresno.ucsf.edu/pediatrics/downloads/edinburghscale.pdf.)There are few controlled studies of the treatment of depression in postpartum patients specifically, so treatment recommendations are based on the general population. A systematic review noted that paroxetine, sertraline, and nortriptyline have not been found to have adverse effects on infants, but fluoxetine should be avoided in breastfeeding women.

A 28-year-old gravida 1 para 1 presents for a routine annual visit. You provided prenatal care for her pregnancy, which was uncomplicated and resulted in a normal vaginal delivery 12 months ago. She just stopped breastfeeding and asks about the best timing for her next pregnancy. With regard to reducing the risk of complications, which one of the following would you recommend as the minimum interval between pregnancies? 6 months 12 months 18 months 24 months 36 months

C Short interpregnancy intervals are associated with an increased risk of low birth weight, prematurity, stillbirth, and uterine rupture in patients with a previous history of cesarean section. The results of one meta-analysis indicated that the optimal interpregnancy interval to reduce the likelihood of low birth weight and premature delivery is 18-56 months (SOR A).

True statements regarding external cephalic version (ECV) include which of the following? (Mark all that are true.) The optimal time to perform ECV is at 35 weeks gestation ECV can appropriately be attempted in a physician's office Fetal monitoring should be performed both before and after attempted ECV Successful ECV significantly reduces the rate of cesarean delivery

C and D According to the American College of Obstetricians and Gynecologists (ACOG), external cephalic version (ECV) carries a low risk of an adverse event, and significantly reduces the rate of cesarean delivery when the fetus is in a breech position. ACOG recommends that the procedure be offered to all women whose fetuses are in a breech position near term (SOR A). The maneuver should not be performed before 36 weeks gestation (SOR B). ECV should be attempted only in a location where cesarean delivery can be readily accomplished if the need arises (SOR C), and fetal monitoring is recommended both before and after attempting ECV (SOR C).

Centers for Disease Control and Prevention guidelines for intrapartum group B Streptococcus (GBS) prophylaxis include which of the following? (Mark all that are true.) If an intrapartum rapid nucleic acid amplification test (NAAT) is negative, no GBS prophylaxis is indicated regardless of the timing of membrane rupture If a woman was GBS positive in her last pregnancy, she should receive antibiotic prophylaxis with her current pregnancy Women with GBS in their urine at any point in pregnancy should receive intrapartum prophylaxis and do not need a GBS culture at 35-37 weeks gestation Women who are GBS positive and have a non-anaphylactic allergy to penicillin should receive cefazolin intravenously for intrapartum GBS prophylaxis Women who are GBS positive with a strain resistant to erythromycin and sensitive to clindamycin, and who have an anaphylactic allergy to penicillin, should routinely be given clindamycin intravenously for intrapartum GBS prophylaxis

C and D The nucleic acid amplification test (NAAT) has sufficient sensitivity and specificity to use in women presenting in labor at term with an unknown group B Streptococcus (GBS) status. If the NAAT is positive, intrapartum prophylaxis is indicated. However, if the NAAT is negative and the patient has a GBS risk factor (rupture of membranes for 18 hours or more, temperature ≥38°C, or gestational age <37 weeks), intrapartum prophylaxis is indicated despite the negative NAAT result.The Centers for Disease Control and Prevention (CDC) guidelines recommend prophylaxis for women who have had a previous child with GBS sepsis or who have GBS in their urine in the current pregnancy. These women should not be screened for GBS at 35-37 weeks gestation, as antibiotics are indicated regardless of the result. Women who were GBS positive in a previous pregnancy need to be screened again, as they may now be GBS negative and would not need intrapartum prophylaxis.Intrapartum GBS prophylaxis is indicated in patients with GBS bacteriuria regardless of when it is detected in pregnancy. Data is lacking on the cost and benefits of screening for GBS at concentrations <100,000 CFU/mL. One poorly designed study found that babies of women who had bacteria at concentrations <10,000 CFU/mL were at increased risk of early onset GBS disease compared with babies of women with no GBS in their urine.GBS-positive women who have a non-anaphylactic allergy to penicillin should receive intrapartum GBS prophylaxis with cefazolin, 2 g intravenous load followed by 1 g intravenously every 8 hours until delivery. GBS-positive women who have an anaphylactic allergy to penicillin should have sensitivities run with their GBS culture. If their GBS is sensitive to erythromycin AND clindamycin, they should receive clindamycin, 900 mg intravenously every 8 hours until delivery; this is a change from earlier guidelines. If their GBS is resistant to erythromycin OR clindamycin, they should receive vancomycin, 1 g intravenously every 12 hours until delivery, for GBS prophylaxis.

Which of the following situations is a clear indication (level A evidence) for administration of anti-D immunoglobulin to an Rh D-negative patient? (Mark all that are true.) A patient at 24 weeks gestation with bleeding from a small abruption A patient undergoing an attempt at external cephalic version at 36 weeks gestation A patient who delivers an Rh-positive infant at 31 weeks gestation, 2½ weeks after receiving 300 µg of anti-D immunoglobulin for prophylaxis A patient who has just had a spontaneous abortion at 11 weeks gestation A patient who suffered blunt abdominal trauma at 26 weeks gestation with no sign of abruption

D Rh-D alloimmunization of Rh-negative pregnant women is a significant health concern, with a 17% risk of sensitization occurring in the absence of adequate passive immunization. While 90% of alloimmunization cases are related to fetomaternal hemorrhage at delivery, there are situations in which the risk of sensitization is high enough to clearly warrant prophylaxis earlier in pregnancy, including first-trimester pregnancy loss and invasive procedures such as chorionic villus sampling, amniocentesis, and fetal blood sampling. While other situations are known to increase the risk of alloimmunization, such as threatened miscarriage, second- and third-trimester bleeding, external cephalic version, and abdominal trauma, the increased risk is not sufficient for a clear, evidence-based recommendation to use anti-D immunoglobulin. Because the half-life of anti-D immunoglobulin is 24 days, women who deliver without risk factors for excessive bleeding within 3 weeks of receiving the 28-week dose do not need to be reimmunized.

A 30-year-old primigravida has diet-controlled gestational diabetes with no other pregnancy complications. Which of the following would be appropriate management? (Mark all that are true.) Antepartum fetal monitoring beginning at 32 weeks gestation Cesarean delivery to prevent birth trauma Frequent blood glucose monitoring during the active stage of labor Screening for type 2 diabetes at 6 weeks post partum and, if indicated, annually thereafter

D The American College of Obstetricians and Gynecologists (ACOG) recommends routine antepartum testing for women with gestational diabetes mellitus (GDM) who require insulin, have poorly controlled glucose levels, or are hypertensive (SOR C). There is no consensus as to whether antepartum testing is indicated in women whose GDM is well controlled with diet or oral medication, although such testing is widely practiced. There is also no consensus as to the optimal antepartum testing regimen.ACOG also recommends consideration of cesarean delivery for women with GDM and a fetus estimated to weigh 4500 g or more, as this may reduce the risk of shoulder dystocia and permanent brachial plexus injury (level of evidence 2). There currently is no indication for routine use of cesarean delivery to prevent birth trauma in women with GDM and an estimated fetal weight <4500 g. However, it is important to keep in mind the considerable margin of error for fetal weight estimation by ultrasonography. Frequent glucose monitoring during labor is indicated for women with insulin-requiring GDM, but not for those with diet-controlled GDM.All women with GDM are at increased risk for developing type 2 diabetes later in life, and should therefore be screened at 6 weeks post partum; if normoglycemic at that time, they should be screened on a yearly basis thereafter (SOR C).

A 28-year-old gravida 2 presents with crampy left-sided pelvic pain that began last night. She had a positive pregnancy test a week ago but has not had any bleeding, fever, or emesis. She had a spontaneous miscarriage at age 24. She has not been using birth control for over 2 years because she thought she could not get pregnant. She is not certain when her last menstrual period started but thinks it was about 6 weeks ago. On examination you confirm diffuse tenderness in the mid- and left-sided pelvic area with no peritoneal signs, no bleeding on speculum examination, and no apparent cervical or vaginal abnormalities.You are concerned that she may have an ectopic pregnancy and order transvaginal ultrasonography, which shows no sign of intrauterine pregnancy and no adnexal mass. Her quantitative β-hCG level is 900 IU/L.What do you advise the patient? (Mark all that are true.) The negative pelvic ultrasonography rules out an ectopic pregnancy The quantitative β-hCG rules out an ectopic pregnancy The combination of the ultrasound findings and the β-hCG level is diagnositic of an anembryonic pregnancy (blighted ovum) It is possible that she has a normal pregnancy or an ectopic pregnancy, or that she will miscarry Dilatation and curettage is indicated to distinguish between an anembryonic pregnancy and an ectopic pregnancy

D When the quantitative β-hCG level is <1500 IU/L and ultrasonography does not show evidence of either an intrauterine or an ectopic pregnancy, it is still possible that a woman is early in a normal pregnancy, an ectopic pregnancy, or a pregnancy that will miscarry. With a β-hCG level of 1500-2500 IU/L, a gestational sac should be seen on transvaginal ultrasonography. When the gestational sac is 10 mm in size, a yolk sac should be seen. A yolk sac confirms an intrauterine pregnancy. Occasionally, a woman may have an intrauterine AND an ectopic pregnancy. This is referred to as a heterotopic pregnancy and occurs in approximately 1 in 30,000 pregnancies.If an intrauterine gestational sac over 18 mm does not have a yolk sac, it is an anembryonic, nonviable pregnancy. Care must be taken to distinguish a gestational sac from a pseudosac. A pseudosac is fluid in the uterus with a tubal pregnancy. A true gestational sac will have two rings around it, known as a double decidual sign. If a fetus measures 5 mm in length it should have a heartbeat. A fetus over 5 mm without a heartbeat is an embryonic demise, a term that should replace the less specific term missed abortion.

A 22-year-old primigravida at 10 weeks gestation is concerned that her partner, who is in the army, will be stationed overseas at the time of her delivery. The partner is scheduled to ship out 2 weeks before her estimated delivery date and will be gone for 6 months. She would like to know if labor can be induced a few days before he leaves. Her pregnancy is complicated by smoking, obesity, and a history of genital herpes, but she is otherwise well. Her dates were confirmed by an ultrasound evaluation at 7 weeks gestation.Which one of the following would be appropriate advice? Smokers should be induced at term to avoid respiratory compromise Obese patients should be induced at 38 weeks gestation to avoid macrosomia She should be induced if she has a herpes outbreak at 38 weeks gestation Unless there is a medical indication, she should not be induced before 38 weeks gestation Unless there is a medical indication, she should not be induced before 39 weeks gestation

E Induction of labor is a common procedure in the United States, with over 22% of pregnant women undergoing induction. Factors in the decision to induce labor should include maternal and fetal medical conditions, gestational age, and cervical status as assessed by the components of the Bishop score (dilation, effacement, station, consistency, and position of the cervix). Indications for induction of labor prior to the spontaneous onset of contractions include pregnancy complications such as abruptio placentae, chorioamnionitis, fetal demise, fetal growth restriction, oligohydramnios, gestational hypertension, preeclampsia, eclampsia, premature rupture of the membranes, and postterm pregnancy. Other indications include maternal medical conditions such as diabetes mellitus, renal disease, chronic pulmonary disease, chronic hypertension, and antiphospholipid syndrome. Patients may also be induced for logistical or psychosocial reasons if they have reached 39 weeks gestation confirmed by ultrasonography prior to 20 weeks gestation, fetal heart tones have been documented for at least 30 weeks by Doppler ultrasonography, or it has been 36 weeks since a positive urine or serum pregnancy test result was documented. There is no evidence that elective induction decreases the rate of macrosomia in obese patients, and active herpetic lesions are a contraindication to induction. Although this patient should be counseled regarding smoking cessation for the health of the fetus and herself, smoking is not an indication for induction.

A 23-year-old gravida 2 para 1 at 39 weeks gestation presents with spontaneous labor and intact membranes. Fetal heart monitoring is reassuring, her contraction pattern is good, and she has advanced from 4 cm dilation to 7 cm dilation over the last 2 hours. She is negative for group B Streptococcus and has had no known complications during her prenatal course or during labor. You are called to the room because her membranes have spontaneously ruptured and thick, particulate meconium is noted.Which of the following is recommended in this situation? (Mark all that are true.) Amnioinfusion with saline during labor Routine oropharyngeal and nasopharyngeal suctioning on the perineum Endotracheal intubation and suction of the vigorous infant after stimulation Endotracheal intubation and suctioning prior to stimulation if the infant is depressed at birth Endotracheal intubation and suctioning if the airway is obstructed

E Meconium aspiration syndrome (MAS) is a significant cause of newborn morbidity in term infants; meconium-stained fluid occurs in 7%-22% of deliveries, and in these cases, the infant develops MAS as a complication between 1.7% and 35% of the time. MAS is thought to occur because of in utero aspiration of meconium-stained fluid, which then causes a chemical pneumonitis in the lungs.While a Cochrane review of the use of amnioinfusion suggests that its use can decrease the incidence of MAS when thick meconium is discovered during labor, the review was based on small studies. A much larger, well-designed randomized, controlled trial showed clearly that if there was continuous electronic fetal monitoring, there was no decrease in MAS, perinatal morbidity, or mortality when amnioinfusion was performed in patients with thick meconium-stained amniotic fluid (SOR B).While endotracheal intubation and suction of the vigorous infant after stimulation were used in the past, this is no longer recommended; a meta-analysis has shown that tracheal intubation and suction in these vigorous infants provides no benefit (SOR A). In addition, new neonatal resuscitation guidelines released by the American Heart Association and the American Academy of Pediatrics in 2015 now recommend against routine endotracheal intubation and suctioning in depressed infants (with poor muscle tone or inadequate breathing efforts) born through meconium-stained amniotic fluid. Instead, the guidelines recommend that the initial steps of resuscitation for these infants be completed under the radiant warmer with positive pressure ventilation initiated if the infant remains depressed after the initial steps are completed (inadequate respirations or heart rate <100 beats/min). The new guidelines also recommend that appropriate interventions to support ventilation and oxygenation be initiated as indicated for each individual infant, including intubation and suction if the airway is obstructed.The more time-honored tradition of suctioning with either a bulb or a DeLee trap on the perineum is referred to in the resuscitation guidelines from the International Consensus Conference and is thought to provide no benefit (SOR C).

In which of the following patients could routine screening for gestational diabetes reasonably be omitted? (Mark all that are true.) A 32-year-old gravida 4 para 3 with a pre-pregnancy body mass index (BMI) of 30 kg/m2 whose last baby weighed 4500 g (9 lb 15 oz) at birth An 18-year-old primigravida with a pre-pregnancy BMI of 27 kg/m2 whose mother has type 2 diabetes A 21-year-old primigravida of Samoan ancestry with a pre-pregnancy BMI of 25 kg/m2 A 40-year-old gravida 2 para 1 with an uncomplicated first pregnancy, a pre-pregnancy BMI of 22 kg/m2, and no family history of diabetes mellitus A 23-year-old primigravida with a pre-pregnancy BMI of 25 kg/m2 and no family history of diabetes mellitus

E Screening all pregnant women for gestational diabetes mellitus (GDM) will detect the most cases, but women who meet ALL of the following low-risk criteria may stand to gain less from screening (SOR C): age less than 25 BMI ≤25 kg/m2 not a member of an ethnic group with an increased prevalence of diabetes mellitus (e.g., Pacific Islanders, Hispanics, Native Americans, African Americans, South or East Asians) no history of abnormal glucose tolerance no history of previous pregnancy complications typically associated with GDM no family history of diabetes mellitus in a first degree relative


Conjuntos de estudio relacionados

Principles of Auditing Chapter 5

View Set

Service Cloud Certification Prep

View Set